Pharma- Neurology (UWORLD)

अब Quizwiz के साथ अपने होमवर्क और परीक्षाओं को एस करें!

42 years woman -History: schizophrenia, brought to clinic by parents after attempting to drink rubbing alcohol in response to voices telling her to kill herself -Patient believes TV newscaster is sending her secret messages and that the devil injects her with poison at night while she is sleeping -First developed symptoms at age of 23 and has had 7 previous psychiatric hospitalization and 2 suicide attempts -Trials of haloperidol, risperidone, and olanzapine have yielded minimal improvement -Conditions: hypertension, mild obesity; medication therapy with clozapine is initiated; what should be regularly monitored? a)Absolute neutrophil count b)Clozapine blood level c)Creatinine level d)ECG e)Liver function f)Platelet count g)Prolactin level h)Thyroid function

answer: Absolute neutrophil count -SGA clozapine is the only antipsychotic that has consistently shown superior efficacy in treatment-resistance schizophrenia and schizophrenia associated with persistent suicidality; clozapine has high affinity for multiple dopamine and serotonin receptors, but precise pharmacological mechanism responsible for its superior efficacy is unknown -Clozapine binds loosely and transiently to dopamine D2 receptors, causing significantly fewer extrapyramidal symptoms compared to FGAs -Neutropenia (<1000 cells/mm3) and potential for life-threatening agranulocytosis are the major adverse effects of clozapine; risk of agranulocytosis is approximately 1%; therefore, treatment requires regular monitoring of the patient's absolute neutrophil count -Treatment should be stopped if neutropenia occurs; seizures and myocarditis are other important adverse effects that require provider vigilance option B= checked after initial target dose is reached, but further dosage adjustments are usually based on clinical response; not regularly monitored Option C and H= should be monitored with lithium due to potential to adverse effects on thyroid and renal function option D= baseline ECG is required and physicians should be alert to development of cardiovascular symptoms suggestive of myocarditis, routine ECGs are not required; among SGAs, ziprasidone is most often noted for causing a prolonged QT interval option E and F= can be decreased with some antipsychotics and anticonvulsants; not regularly monitored with clozapine option G= risperidone has highest risk of prolactin elevation

72 years man -History: Parkinson disease (follow-up) -Medication: levodopa/carbidopa (5 years; required increasing dose to control symptoms; currently takes maximum dose) -Past months: bothersome motor fluctuations, worsening stiffness and difficulty moving before his next scheduled dose (improves after medication) -Entacapone is added to treatment regimen; mechanism? a)Decreasing peripheral levodopa degradation b)Directly stimulating dopamine receptors c)Enhancing the effect of endogenous dopamine d)Inhibiting central muscarinic receptors e)Inhibiting centrally-active acetylcholinesterase

answer: Decreasing peripheral levodopa degradation -Levodopa is the immediate precursor of dopamine, a neurotransmitter that is absent in the nigrostriatum of patients with Parkinson disease; Dopamine itself can't be administered directly due to its inability to cross the BBB, however, levodopa can cross this barrier -Levodopa is rapidly metabolized in the peripheral tissues to dopamine (via DOPA decarboxylase) and 3-O-methyldopa (via catechol-O-methyltransferases (COMT)); therefore, only a small percentage of levodopa reaches the brain (Consequently, levodopa is typically administered with carbidopa, a peripheral DOPA-decarboxylase inhibitor, but even then, only 5-10% of levodopa reaches the brain) -Entacapone is a COMT inhibitor that increases the bioavailability of levodopa by inhibiting its peripheral methylation; Combining levodopa with entacapone helps increase the plasma half-life of levodopa, producing more stable levodopa plasma concentration and prolonging the therapeutic effect of each dose -COMT inhibitors are currently used to treat Parkinson disease patients with motor fluctuations who are experiencing end-dose "wearing-off" periods with levodopa/carbidopa therapy (COMT inhibitors should only be used with levodopa, otherwise they are ineffective) option B= dopamine agonists (e.g., bromocriptine, pramipexole, ropinirole) preferentially stimulate dopamine 2 receptors option C= amantadine likely enhances the effect of endogenous dopamine by increasing dopamine synthesis/release and inhibiting the reuptake of dopamine option D= anticholinergics such as trihexyphenidyl and benztropine inhibit central muscarinic receptors; these drugs are mainly used for drug-induced parkinsonism, but patients with tremor as the predominant symptom also respond well option E= acetylcholinesterase inhibitors (e.g., donepezil, rivastigmine) increase the amount of Ach in the CNS; commonly used in patients with dementia to increase cholinergic activity in the prefrontal cortex; should be used cautiously in patients with Parkinson disease as higher levels of Ach in the basal ganglia can exacerbate symptoms

72 years woman -Forgetfulness (misplaces items and quickly forgets); progressively worsened over past 2-3 years -Month ago= became lost while driving in the neighborhood where she lived most of her adult life -Exam: short-term memory deficit with no other significant neurologic findings; oral cholinesterase inhibitor is prescribed -Effect of drug? a)Delayed disease progression due to neuroprotective effect b)Improved cognitive function with no change in disease course c)Neuroprotection with no effect on cognitive performance d)Reversal of cognitive decline due to increased neuroplasticity e)Symptomatic improvement only through the placebo effect

answer: Improved cognitive function with no change in disease course -Insidious onset of increasing forgetfulness, visuospatial deficits (e.g., misplacing objects), and executive dysfunction (e.g., getting lost in familiar surroundings), coupled with normal neurologic examination= Alzheimer disease -Degeneration of cholinergic neurons= reduced Ach synthesis in basal forebrain, contributes to memory loss and executive dysfunction that characterized AD; Cholinesterase inhibitors (e.g., donepezil, rivastigmine, galantamine) block acetylcholinesterase in synaptic cleft to compensate for loss of cholinergic transmission -Provide symptomatic improvement, but don't enhance neuron survival; therefore, they are not considered neuroprotective -In contrast, NMDA receptor antagonists (e.g., memantine) inhibit glutamate NMDA signaling to prevent excitotoxicity (calcium overload) and subsequent neuronal apoptosis (neuroprotective, slow down progression)= choice A option C= antioxidants, e.g., vitamin E, may delay progression of AD through neuroprotective effect, but don't improve cognition option D= neuroplasticity, the ability of neurons to reorganize or restructure, is most notably prominent in childhood; reduced neuroplasticity has been implicated in multiple neuropsychiatric conditions, e.g., psychosis

3 years girl -Leg stiffness; patient is clumsy walker and requires assistance to ambulate -Born preterm and has delayed developmental skills -Exam: increased knee and ankle reflexes and clonus at ankles bilaterally -Muscle tone of upper extremities and neck is mildly increased -MRI= periventricular leukomalacia suggestive of cerebral palsy -Oral medication prescribed that reduces extremity stiffness; mechanism? a)Activation of NMDA glutamate receptors b)Agonistic activity at GABA receptors c)Blockade of acetylcholine muscarinic receptors d)Blockade of receptors for glycine e)Increased dopaminergic activity

answer: Agonistic activity at GABA receptors -Patient has cerebral palsy, condition characterized by permanent, nonprogressive motor dysfunction caused by damage to developing brain -Certain subtypes (e.g., spastic cerebral palsy) often manifest as delayed gross motor milestones and spasticity (e.g., hypertonia, hyperreflexia) -Spastic cerebral palsy may be caused by white matter necrosis (seen in this patient as periventricular leukomalacia), which leads to a loss of descending inhibitory control from upper motor neurons= results in muscle overactivity, leading to increased tone and hyperreflexia -Enhancement of CNS inhibition (treatment)= enhancing GABA= 1) GABA-A receptor mediates fast response to GABA because it is an ion-gated chloride channel; activation= leads to rapid muscle relaxation and sedation; its activity is enhances by BZs which can be used to treat spasticity 2) GABA-B receptor mediates slower response to GABA because it is mediated by G protein that opens potassium channels; it is agonized by baclofen (can be used to treat spasticity) option A= would worsen spasticity option C= in drug induced parkinsonism, or rigidity in parkinson option D= glycine is an inhibitory neurotransmitter found in the spinal cord not in brain; blockage of glycine receptors (e.g., strychnine poisoning)= increased muscle contraction option E= helps with parkinson symptoms

23 years woman -Restlessness in legs and inability to lie or sit still -Diagnosed with schizophrenia a month ago (medication initiated) -Dose increased after 2 weeks, says she haven't heard voices after medication increased; BP is high; patient is alert, oriented, fidgety and anxious; diagnosis? a)Acute dystonia b)Akathisia c)Drug-induced parkinsonism d)Neuroleptic malignant syndrome e)Psychotic agitation f)Tardive dyskinesia

answer: Akathisia -Experiencing akathisia, type of extrapyramidal side effect associated with antipsychotics -Ranges from mild subjective feelings of tension to marked physical restlessness (distressing); presents days-weeks after treatment or increasing dose (inability to sit or stand in one position); treatment= deceasing antipsychotic dose or if feasible with beta blocker or BZs option A= extrapyramidal side effect; acute onset of involuntary muscle contraction of neck, mouth, tongue or eye muscles; occurs within hours to days of antipsychotic use, especially with rapid dose escalation of high potency, FGA, e.g., haloperidol and is treated with anticholinergics like diphenhydramine or benztropine option C= extrapyramidal side effect= cogwheel rigidity, masked facies, bradykinesia, tremor, decreased arm swing; treatment with anticholinergics, e.g., benztropine option D= acute, potentially fatal reaction to antipsychotics; hyperthermia, sympathetic hyperactivity (e.g., tachycardia, diaphoresis), severe "lead-pipe" rigidity, and altered mental status option E= auditory hallucinations improved; increase dose if present option F= involuntary and potentially irreversible movement disorder due to prolonged antipsychotic exposure (typically years); lip smacking, biting, grimacing, tongue protrusions, choreoathetoid movements of head, limbs and trunk

-Medical student observing experiment -Drug A is being IV administered to a pregnant dog; HR, BP, pupil size and uterine contractions are recorded -After drug A infusion= uterine contractions decrease and mydriasis -Drug A? a)Beta-adrenergic agonist b)Alpha and beta-adrenergic agonist c)Alpha-adrenergic agonist d)Beta-adrenergic antagonist e)Alpha-adrenergic antagonist and beta-adrenergic agonist

answer: Alpha and beta-adrenergic agonist -Pupillary dilator muscle of eye possesses alpha-1 receptor and stimulation of these receptors results in contraction of muscle with subsequent dilation of pupil (mydriasis); ophthalmic preparation of phenylephrine (selective alpha-adrenergic agonist) is used to dilate pupil before ophthalmoscopic exam -Uterus contains beta-2 receptors= stimulation= uterine relaxation (tocolysis) which is used in obstetrics to defer premature labor (ritodrine, terbutaline used for this purpose)

65 years man -New-onset confusion, suprapubic discomfort, lack of urine output -History: benign prostatic hyperplasia, hypertension, hyperlipidemia, T2D complicated by neuropathy -Also has chronic insomnia (little benefit with medication); exam: afebrile, confused, oriented only to self; suprapubic fullness is present -Urinary catheter is placed and immediately drains 1000ml of urine -Medication that contributed to patient's current condition? a)Amitriptyline b)Atorvastatin c)Duloxetine d)Finasteride e)Gabapentin f)Metformin g)Tamsulosin

answer: Amitriptyline -TCAs (e.g., amitriptyline)= used for insomnia or adjunctive pain management when other medications are not effective -TCAs have strong anticholinergic effects (i.e., confusion, constipation, and acute urinary retention) -Elderly patients are at increased risk for side effects due to comorbid conditions (e.g., dementia, benign prostatic hyperplasia), decreased hepatic and renal clearance of medications, and a higher burden of concurrent medications that can interact with TCA's and contribute to adverse effects (TCAs are relatively contraindicated in elderly) option B= hepatitis, myositis option C= SNRI; for depression and anxiety and chronic pain disorders; can cause confusion but rarely causes obstructive voiding symptoms option D= used for treatment of benign prostatic hyperplasia and should not cause urinary obstruction or retention option E= used for painful diabetic neuropathy; can cause sedation, confusion, incoordination; but doesn't have significant anticholinergic side effects option F= lactic acidosis option G= used for BPH

58 years man -Presents with lower extremity pain; patient began experiencing tingly discomfort in toes a year ago (especially at night, when sheets touched his skin) -Symptoms have progressed to burning and stabbing pain in feet and lower legs -Diagnosed: T2D (8 years ago) and hypertension (3 years ago) -Lower extremity exam: normal appearing skin; touching feet elicits sensation of burning pain -Loss of vibration sensation in toes and ankle reflex is absent bilaterally -Best initial medication for symptoms? a)Amitriptyline b)Clonazepam c)Fluoxetine d)Naproxen e)Oxycodone f)Pyridoxine

answer: Amitriptyline -long-standing diabetes mellitus has tingling/burning pain in feet, loss of vibration sensation and loss of ankle reflexes= distal symmetric polyneuropathy (diabetic neuropathy) -Long-standing diabetes mellitus is often associated with damage to the distal axons of sensory neurons due to accumulation of cross-linked glycosylated serum proteins in the vasa nervorum, which causes inflammation, vasoconstriction, and neuronal oxidative stress -Initial symptoms: tingling, numbness, burning pain in toes and/or feet; sensation to pain, light touch, temperature, vibration, and proprioception ad diminished in affected area; Babinski and ankle reflex are often absent Treatment: glycemic control and foot care to limit development of ulcers and need for amputation; patients with painful neuropathy also receive SNRIs (e.g., duloxetine, venlafaxine), a gabapentinoid (e.g., pregabalin, gabapentin) or TCA such as amitriptyline (modulates pain transmission by inhibiting voltage gated sodium channels in sensory nerves, blocking NMDA receptors in the spinal cord, and altering NE signaling in the brain) option B= BZs often used for anxiety, muscle relaxation or nausea option C= SSRIs for depression and anxiety option D= NSAIDs are used for acute painful conditions such as back pain, muscle strain, or bone fracture option E= opioids are not recommended in neuropathic pain due to questionable efficacy and risk of dependence option F= patients taking isoniazid require pyridoxine supplementation to prevent development of drug-induced peripheral neuropathy

65 years man -Increasing muscle rigidity and gait instability -Diagnosed: parkinson (2 years ago); treated with selegiline -Father= parkinson; mother= stroke -Switched to levodopa to control patient's symptoms; side effects that persist after adding carbidopa to levodopa treatment? a)Nausea and vomiting b)Tachyarrhythmias c)Hot flashes d)Postural hypotension e)Anxiety and agitation

answer: Anxiety and agitation -Anxiety and agitation are central effects of dopamine and are caused by L-DOPA, regardless whether carbidopa is added or not (increases, due to more dopamine reaching brain) -Adding carbidopa= inhibits peripheral conversion of levodopa= more dopamine reaches brain= anxiety, agitation, insomnia, confusion, delusions, confusion, hallucinations (reducing dosage helps with symptoms, if you can't, add atypical antipsychotics like clozapine) option A, C, D= peripheral symptom of dopamine; carbidopa inhibits it option B= peripheral formation of catecholamines from levodopa

35 years man -Hospitalized for psychotic episode in which he heard voices of God and the devil and believed that his family was plotting to kill him -Improved rapidly with medication therapy and was discharged -3 weeks later comes to ED due to generalized muscle stiffness and shaking of his right hand -Temperature, BP, and pulse are normal; alert, oriented with mild paranoia but no auditory hallucinations -Best treatment of current symptoms? a)Benztropine b)Dantrolene c)Diazepam d)Haloperidol e)Levodopa f)Propranolol

answer: Benztropine -Psychotic episode treated with antipsychotic= caused drug-induced parkinsonism (EPS); symptoms: rigidity, resting/postural tremor, masked facies and bradykinesia (occurs within first 3 months of drug initiation) -Other extrapyramidal side effects: acute dystonia (sudden, sustained contraction of muscle groups) and akathisia (restlessness and inability to sit still) -Drug-induced parkinsonism= caused by dopamine D2 receptor blockade in the nigrostriatal pathway; if antipsychotic can't be stopped or reduced; can be treated with centrally acting anticholinergic agent (e.g., benztropine) option B= direct acting muscle relaxant used in treatment of neuroleptic malignant syndrome; NMS presents with diffuse muscle rigidity, high fever, sympathetic hyperactivity (e.g., hypertension, tachycardia) and altered mental status) option C and F= BZs and beta blockers can be used in treatment of akathisia; characterized by subjective restlessness option D= FGA, potent D2 blocker= exacerbate condition option E= exacerbates psychosis

23 years woman -Chest pain, dyspnea, palpitations, perioral numbness and sweating (sudden) -Extremely anxious, similar episodes 2-3 times a week (several months) -High BP, pulse and RR; normal cardiac enzymes and ECG -Diagnosed with panic disorder; receives medication and feels better within an hour; mechanism of action of medication? a)Affects G protein signal transduction b)Alters GABA metabolism c)Binds allosterically to the GABA receptor d)Competes with GABA at its binding site e)Physically blocks the ion channel lumen f)Upregulates GABA receptor density

answer: Binds allosterically to the GABA receptor -Classic symptoms of panic attack (abrupt and unexpected onset of characteristic physical symptoms and negative medical workup); she most likely receive a benzodiazepine (used in emergency settings commonly because it provides rapid relief of acute anxiety) -BZs work at the GABA-A receptor complex; GABA is the main inhibitory neurotransmitter in the CNS and is synthesized from glutamate using glutamate decarboxylase -GABA-A receptor complex consists of 5 subunits and a central chloride ion channel, with different binding sites for GABA and various drugs (e.g., BZs, barbiturates) -Benzodiazepines act by binding to the BZ binding site, which allosterically modules the binding of GABA, resulting in increased frequency of chloride channel opening -Influx of chloride ions into the neurons causes neuronal hyperpolarization and inhibition of AP; barbiturates act at a separate allosteric site and work by prolonging the duration of channel openings in response to GABA, and, at higher doses, as GABA-A receptor agonists option A= Baclofen is an agonist as GABA-B, which is attached to a G protein option B= valproic acid and vigabatrin function to reduce GABA catabolism option F= prolonged BZ use has been shown to cause downregulation of GABA-A receptors= BZ tolerance

10 years boy -Fall while riding his bicycle; hit his forehead on concrete road barrier and sustained laceration but did not lose consciousness -No headache, nausea, vomiting, vision disturbances, or focal weakness or numbness -Received tetanus vaccine at 6 years; exam: 2cm laceration above left eyebrow; no other injuries -Prior to laceration repair with suture, lidocaine is injected in subcutaneous tissue around laceration (loses sensation and pain) -Effects are due to? a)Blockade of potassium channels b)Blockade of sodium channels c)Decreased calcium current d)Increased chloride conductance e)Interruption of axonal transport

answer: Blockade of sodium channels -Lidocaine is a local anesthetic that interrupts nerve fiber transmission by blocking sodium channels in the neuronal cell membrane, preventing depolarization of the nerve; local anesthetics have a greater effect on small myelinated nerves, so they preferentially inhibit neurons that carry pain and temperature sensation, pressure sensation and motor functions are often preserved at typical doses -Local anesthetics are weak bases that exist in charged and uncharged forms; when in the uncharged form, they diffuse through the cell membrane to block sodium channels from the inside of cell -Increased acidity in the surrounding tissue, which occurs in infected wounds, results in more anesthetic molecules remaining in charged form, rendering them unable to penetrate cell membrane to exert an effect (raise dose)

67 years man -Acute worsening of chronic back pain associated with new-onset bilateral leg weakness and numbness -History: colorectal cancer with vertebral metastases for which he has received colectomy, chemotherapy and radiation treatment -Patient takes oral morphine sulfate for back pain; exam: focal vertebral tenderness, along with reduced muscular strength, decreased sensitivity to light touch and spasticity in both legs -Imaging= vertebral compression fracture with cord compression -Surgical decompression is planned; which of the following drugs may precipitate opioid withdrawal symptoms is used in this patient? a)Buprenorphine b)Codeine c)Fentanyl d)Hydromorphone e)Meperidine f)Methadone g)Oxymorphone

answer: Buprenorphine -Opioids work by selectively binding to G protein-coupled opioid receptors, mimicking the actions of endogenous opioid peptides -However, long term activation of mu-opioid receptors on nociception-transmitting neurons is associated with increased pain sensitivity (central sensitization) due to: 1) increased turnover of inhibitory opioid receptors (receptor downregulation) and decoupling of receptors from their second messenger system (receptor decoupling) 2) upregulation of excitatory NMDA receptors -This manifest as an increasing dose requirement to provide the same level of pain relief (tolerance) and pain sensations triggered by benign stimuli (opioid-induced hyperalgesia) -Opioid withdrawal can occur 24-48 hours of acute cessation in opioid-tolerant patients and is characterized by nausea/vomiting, myalgias, dilated pupils, diaphoresis and tachycardia -Buprenorphine is a partial opioid agonist that has low intrinsic activity (efficacy) for opioid mu-receptors; however, it binds with high affinity (potency) and can prevent binding of other opioid medications; therefore, buprenorphine acts as an opioid receptor antagonist in the presence of full opioid agonists and can precipitate withdrawal in opioid-tolerant patients with chronic pain

43 years man -Depression; feels sad and unmotivated, sleeps 12 hours a day and has increased appetite, poor concentration at work, and fleeting thoughts of suicide -Major depressive disorder is diagnosed and antidepressant medication administered -Several weeks after medication= depressive symptoms mildly improved, but now distressed by significant decrease in libido and impaired sexual performance -Considering stopping medication for alternate treatment; appropriate drug? a)Bupropion b)Citalopram c)Imipramine d)Sertraline e)Trazodone f)Venlafaxine

answer: Bupropion -This patient was most likely treated with an SSRI, a commonly prescribed initial treatment for major depressive disorder that causes sexual dysfunction in up to 50% of patients -SSRI-related sexual side effects include decreased libido, decreased arousal, anorgasmia, and increased ejaculation latency -Bupropion ("keeps the penis on") is an antidepressant that does not cause sexual dysfunction -Classified as NE-dopamine reuptake inhibitor; because of stimulating effects, bupropion is useful in treating depression associated with hypersomnia and low energy -Also preferred because it causes less weight gain compared to other antidepressants -Contraindicated in patients with bulimia nervosa, anorexia nervosa, and seizure disorders because it lowers seizure threshold option B and D= SSRI, cause sexual dysfunction option C= TCA with numerous side effects including sexual dysfunction; dangerous in overdose because of cardiotoxicity, not first-line in depression option E= highly sedative antidepressant that can be helpful for patients with insomnia; can cause sexual dysfunction and priapism option F= SNRI; causes sexual side effects

30 years man -Sudden onset severe headache and confusion -History: autosomal dominant polycystic kidney disease -Neurology exam: no meningeal signs or focal neurological deficits -CT (head)= blood in subarachnoid space -Admitted to critical care unit for further management -5th after admission, experiences new onset weakness in right arm and leg; medication that would have prevented this complication? a)ACE inhibitor b)Beta-adrenergic blocker c)Calcium channel blocker d)Central sympatholytic e)Nitrate f)Osmotic diuretic g)Thiazide diuretic

answer: Calcium channel blocker -Subarachnoid hemorrhage developed new-onset focal neurological deficits 5 days after initial symptoms, a presentation characteristic of delayed cerebral ischemia due to cerebral vasospasm; SAH is most often caused by rupture of a saccular (berry) aneurysm, which occurs frequently in patients with autosomal dominant polycystic disease -Cerebral vasospasm with subsequent ischemia is most common complication of SAH and typically manifests 3-12 days after initial incident with a change in mental status and/or new focal neurologic deficits -Nimodipine, a selective calcium channel blocker, improves outcomes in patients with SAH by preventing or reducing the extent of ischemic neurological deficits caused by cerebral vasospasm -The neuroprotective effect is likely due to combination of increased cerebral vasodilation and decreased calcium-dependent excitotoxicity option A and B= Enalapril and labetalol can be used to control hypertension in patients with SAH= reduces rebleeding; but rebleeding usually occurs in first 24 hours after SAH option D= e.g., methyldopa, clonidine stimulate alpha-2a receptors centrally, resulting in decrease in generalized sympathetic outflow and reduced BP; no role in prevention ischemic effects due to vasospasm option E= e.g., methyldopa, clonidine stimulate alpha-2a receptors centrally, resulting in decrease in generalized sympathetic outflow and reduced BP; no role in prevention ischemic effects due to vasospasm option F= e.g., mannitol, part of treatment for increased ICP, hydrocephalus can be a complication of SAH and result in elevated ICP, it typically presents with gradual change in level of consciousness accompanied by ocular changes (e.g., downward eye deviation, miosis) rather than focal extremity deficits option G= first-line in essential hypertension; not indicated in complications of SAH

38 years woman -Sudden onset episodes of severe, right sided facial pain -"knife-stabbing my face", lasts several seconds and usually instigated by a meal or teeth brushing; no nausea or vomiting; denies visual changes -History: hyperlipidemia; drinks alcohol socially -No family history of facial pain; best initial treatment? a)Baclofen b)Haloperidol c)Diazepam d)Carbamazepine e)Propranolol f)Phenobarbital g)Valproic acid

answer: Carbamazepine -Trigeminal neuralgia (tic douloureux)= episodes of sudden and severe pain in distribution of CN V (particularly V2 and V3)= described as "stabbing" or like an "electric shock", it is usually unilateral and doesn't last more than few seconds, but occurs repeatedly -Pain triggered by any stimulus to CN V, for example, chewing, teeth brushing, shaving, washing face, swallowing, exposure to hot or cold temperatures -Occur for several months, majority of cases aren't caused by structural abnormality (pathogenesis unclear) -Carbamazepine is the drug of choice; effective in pain reduction in 80% of patients -Like phenytoin, it reduces the sodium channels ability to recover from inactivation -Carbamazepine can cause aplastic anemia, so CBC should be monitored regularly; it is a P450 inducer that increases metabolism of other drugs option A and G= and valproic acid can be used in trigeminal neuralgia, not first-line option B= neuroleptic medication used for treatment of schizophrenia, acute psychoses, acute mania, and Tourette syndrome option C= BZ; used in status epilepticus option E= migraine prophylaxis, reduction in portal venous pressure to prevent variceal bleeding and essential tremors option F= barbiturate used in generalized tonic-clonic seizures

36 years man -Fluorinated inhaled anesthetic (isoflurane) is used to achieve the desirable depth of CNS depression (for surgery) -Increase in which of the following parameters will happen during anesthesia? a)Glomerular filtration rate (GFR) b)Effective renal plasma flow c)Left ventricular ejection fraction d)Hepatic blood flow e)Cerebral blood flow

answer: Cerebral blood flow -General anesthesia encompasses: loss of consciousness, analgesia, amnesia, skeletal muscle relaxation and inhibition of reflexes -Most inhalational anesthetics, barbiturates, and benzodiazepines achieve CNS depression by increasing inhibitory action of GABA -inhaled anesthetics also lock potassium channels in hyperpolarized state Inhalational anesthetics can also affect other organs in the body -Cardiovascular effects: myocardial depression (decrease CO and increase atrial & ventricular pressures); hypotension due to decrease in CO -Respiratory: respiratory depressants (except nitrous oxide); decrease tidal volume and minute ventilation (hypercapnia); suppress mucociliary clearance= predispose to postoperative atelectasis (halothane and sevoflurane have bronchodilation properties= preferred in asthma) -Brain: decrease vascular resistance and lead to increase in cerebral blood flow (undesirable= increases ICP) -Kidneys: decrease GFR, increase renal vascular resistance and decrease renal plasma flow -Decrease hepatic blood flow

44 years man -Acute cholecystitis (hospitalized); being evaluated for anxiety and agitation -Underwent open cholecystectomy without having any operative complications -2 days after admission= patient experiences anxiety, tremulousness and becomes irritable, severely agitated, and verbally abusive to the nursing staff -Drinks 6-8 beers daily for last several years; BP and pulse are high -Exam: hand tremors bilaterally, most appropriate treatment? a)Baclofen b)Carbamazepine c)Chlorpromazine d)Haloperidol e)Chlordiazepoxide f)Phenobarbital g)Propranolol

answer: Chlordiazepoxide -Acute onset of tremulousness, agitation, elevated pulse and BP within 48 hours of admission is suggestive of alcohol withdrawal -Alcohol is a CNS depressant that binds GABA-A receptor complex, enhancing the inhibitory action of GABA -Abrupt cessation of alcohol decreases inhibitory tone and results in CNS excitation; alcohol withdrawal symptoms typically start within 8-12 hours after last drink and include insomnia, tremulousness, anxiety, and autonomic hyperactivity (variable BP, diaphoresis, and tachycardia); alcohol withdrawal seizures can occur within 12-48 hours, and delirium tremens (disorientation, severe agitation, fever) typically begins within 48-96 hours -Alcohol, barbiturates, and BZs have similar effects on GABA receptors and act by enhancing GABA inhibitory action; BZs (e.g., lorazepam, diazepam, chlordiazepoxide) are used as first-line therapy for psychomotor agitation associated with alcohol withdrawal and to prevent progression to seizures and delirium -Longer-acting BZs and those with active metabolites are preferred in the majority of patients due to self-tapering effects, resulting in a smoother course of withdrawal option A= agonist of GABA-B receptors and is used to treat spastic conditions option B= role in outpatient management of mild alcohol withdrawal; insufficient in moderate to severe alcohol withdrawal option C and D= antipsychotic medications that lower seizure threshold; shouldn't be used option F= barbiturate anticonvulsant that enhances GABA activity; no longer used as first-line treatment for alcohol withdrawal because it has worst safety profile than BZs option G= can treat tremor, tachycardia, and hypertension, but doesn't prevent development of seizures or delirium tremens

36 years woman -Mood disorder follow up -History: several depressive episodes that started in her 20s -2 years ago, she was hospitalized after staying up for several nights without sleeping and claiming that she had special powers to "end global poverty and climate change" -Symptoms responded well to treatment and she has remained on same drug regimen since then; however, the patient now has new-onset constipation, dry skin, hair loss -Concerned about weight gain over last 3-4 months despite healthy, low calorie diet; pulse is low -Patient is tired, but physical exam is normal; diagnosis is established and current symptoms due to adverse effect of her drug; which drug? a)Carbamazepine b)Citalopram c)Clozapine d)Lamotrigine e)Lithium f)Risperidone g)Trazodone

answer: Lithium -Weight gain, hair loss, mild bradycardia are most likely due to lithium induced hypothyroidism; lithium is a mood stabilizer primarily used in treatment of bipolar disorder; it can cause hypothyroidism by interfering with normal synthesis and release of thyroid hormone; compensatory increase in TSH release by pituitary can lead to goiter in some patients; lithium-induced hypothyroidism is treated with standard levothyroxine (T4) replacement and doesn't require discontinuation of lithium -Long-term lithium therapy is also associated with adverse effects on renal function (e.g., nephrogenic diabetes insipidus, chronic tubulointerstitial nephropathy), requiring regular monitoring of blood lithium levels and renal function (blood urea nitrogen and creatinine); lithium use during pregnancy has been associated with Ebstein anomaly of the tricuspid valve, but absolute risk is small option A and D= anticonvulsants that are also used as mood stabilizers in bipolar disorder; carbamazepine can cause aplastic anemia; lamotrigine can cause benign rash and Stevens Johnson syndrome option B= SSRI= antidepressant; sexual dysfunction is most common side effect; not used in bipolar disorder option C= SGA indicated for treatment-refractory schizophrenia, associated with agranulocytosis and requires regular monitoring of absolute neutrophil count; can cause weight gain and constipation, not first-line in bipolar disorder and wouldn't explain patient's other hypothyroid features of hair loss, dry skin and bradycardia option F= SGA that can cause weight gain and hyperprolactinemia (e.g., galactorrhea, amenorrhea); not associated with hypothyroidism option G= antidepressant that has significant sedating effects and is most commonly used to treat depression-related insomnia; adverse effects: priapism, anticholinergic effects, like dry mouth and constipation

65 years man -Rehabilitation after stroke; 10 days ago, presented to ED with abrupt onset of left upper and lower extremity weakness; history: hypertension, hypercholesteremia, GERD, hypothyroidism -High BP; weakness and increased tone in left upper and lower extremities; prescribe diazepam to this patient in order to decrease muscle spasticity of involved extremities -Cautioned to avoid which drug? a)Chlorpheniramine b)Loratadine c)Acetaminophen d)Ranitidine e)Lansoprazole f)Folic acid

answer: Chlorpheniramine Diazepam is a long-acting BZ indicated as follows: -As an anxiolytic, it is effective for management of generalized anxiety disorder and panic attacks -The sedative-hypnotic effect of diazepam is useful for short-term treatment of insomnia -As an anticonvulsant, diazepam is indicated for status epilepticus (first-line agent along with chlordiazepoxide for treatment of seizures associated with alcohol withdrawal) -As a muscle relaxant, diazepam can stop spasticity caused by upper motor neuron disorders (multiple sclerosis, strokes, spinal cord trauma) and tetanus Most common side effect: sedation; may also impair coordination and balance, decrease memory and concentration, produce confusion (amplified by co-administration of other CNS-depressants); avoid use with alcohol, barbiturates, neuroleptics, or 1st generation antihistamines From listed, only chlorpheniramine causes sedation and should not be used with diazepam (1st generation antihistamine that acts by blocking both central and peripheral H1 receptors) Chlorpheniramine and other 1st generation antihistamines can be very sedating because they easily penetrate the BBB and accumulate in the CNS; most sedating= diphenhydramine (Benadryl), promethazine, and hydroxyzine Despite sedation, they are used to prevent/treat allergic reaction, motion sickness and as antiemetics option B= 2nd generation antihistamine that blocks peripheral H1 histamine receptors; doesn't enter CNS option D= H2 receptor antagonist that inhibits gastric acid secretion; slight affinity to CYP450 enzyme, but doesn't interact with diazepam option E= PPI that decreases gastric acid secretion; both diazepam and lansoprazole are metabolized by the P450 system, no significant interactions with diazepam

23 years man -Prolonged paralysis following general anesthesia -Anesthesia was induced with propofol, succinylcholine was used for intubation, and anesthesia was maintained via propofol and fentanyl infusions -Patient has not had a return of neuromuscular function after several hours -Mother then discloses she also has been slow to wake up from anesthesia -Reason for prolonged paralysis? a)Decreased first-pass metabolism b)Decreased plasma hydrolysis c)Increased binding to plasma proteins d)Increased redistribution to fat e)Upregulation of target receptors

answer: Decreased plasma hydrolysis -Developed prolonged muscle weakness (e.g., failure of spontaneous respiration, voluntary muscle activity) after receiving succinylcholine most likely has pseudocholinesterase deficiency, an autosomal recessive disorder caused by a genetic polymorphism in the BCHE gene -Succinylcholine is a depolarizing neuromuscular blocking agent used to induce skeletal muscle relaxation during intubation and surgery; it acts as a competitive agonist of nicotinic acetylcholine receptors of the motor endplate, where it induces persistent depolarization, leading to desensitization and skeletal muscle paralysis -Succinylcholine is rapidly hydrolyzed by plasma pseudocholinesterase, only around 10% of administered dose reaches the neuromuscular junction, where it typically has a duration of action of <10 mins -Patients with pseudocholinesterase deficiency are unable to metabolize succinylcholine, which leads to large amount of drug reaching neuromuscular junction. Heterozygotes may experience double normal duration of paralysis; homozygotes can have persistent paralysis for several hours option A= succinylcholine is given IV, it is primarily inactivated in plasma by pseudocholinesterase rather than liver option C= increased binding to proteins would decrease effect of medications option D= propofol is highly lipid soluble; reason for its short duration of action= seconds; activates GABA-A in brain option E= propofol is highly lipid soluble; reason for its short duration of action= seconds; activates GABA-A in brain

75 years woman -Worsening forgetfulness (past few years), difficulty remembering names of people she meets and frequently misplaces household items; recently forgot to turn off stove after cooking -Disrupted sleep with frequent awakenings throughout night -Exam: alert and cooperative but disoriented to time and can recall only 1/3 items on short-memory test -MRI= diffuse cortical atrophy; medication with which mechanism would be useful? a)Decreased presynaptic norepinephrine reuptake b)Decreased synaptic acetylcholine degradation c)Increased postsynaptic NMDA receptor activity d)Increased presynaptic dopamine synthesis e)Increased serotonergic neurotransmission

answer: Decreased synaptic acetylcholine degradation -patient's progressive memory loss, functional impairments (i.e., trouble with activities of daily living), and sleep disturbances are consistent with a dementia syndrome -MRI of head= diffuse cortical atrophy (no evidence of structural disease; e.g., hydrocephalus, chronic subdural hematoma, vascular infarction)= Alzheimer disease (most common primary dementia) -Pathogenesis (complex)= degeneration of cholinergic neurons in basal forebrain contributes to memory loss and functional decline; send projections to hippocampal formations (memory); frontal cortex (executive function), which are impaired in AD -Cholinesterase inhibitors (e.g., donepezil, galantamine) block acetylcholinesterase in synaptic cleft to compensate for loss of cholinergic transmission (modest effects on memory and cognition to improve daily functioning); Anticholinergic medications (e.g., TCAs) should be discontinued -Procholinergic agents improve symptoms (no effect on neuronal survival); in contrast, NMDA receptor antagonists (e.g., memantine) inhibit glutamate NMDA signaling (choice C) to prevent excitoxicity (calcium overload) and subsequent neuronal apoptosis (neuroprotective= slow down progression of disease) option A= norepinephrine-dopamine reuptake inhibitors (e.g., methylphenidate, atomoxetine) are central stimulants; can promote wakefulness to treat narcolepsy or enhance concentration to treat attention deficit hyperactivity disorder option D= dopamine precursors, e.g., levodopa, can enhance dopamine availability in Parkinson, which can be associated with dementia in later stages; used to improve motor rigidity but no role in PD-related cognitive decline option E= SSRIs, e.g., citalopram, first-line medications for major depressive disorder, mimic dementia (i.e., pseudodementia); SSRIs are also used to manage the depressive mood symptoms related to AD

21 years man -Brought to ED (by mom)= found him violently shaking on kitchen floor -Urinated himself and bit his tongue during incident; history: posttraumatic epilepsy (noncompliant with antiseizure medications) -In hospital= several brief, back-to-back generalized tonic-clonic seizures and never fully regains consciousness between episodes; IV lorazepam 2mg (administered) and seizure stops -IV infusion of another drug is started simultaneously to prevent seizure recurrence -Mechanism of 2nd drug infusion? a)Decreases calcium current in thalamic neurons b)Decreases sodium current in cortical neurons c)Dose-dependent activation of postsynaptic GABA-A receptors d)Increases postsynaptic chloride influx e)Irreversible inhibition of GABA catabolism in the central nervous system

answer: Decreases sodium current in cortical neurons -Status epilepticus is a single seizure lasting > 5mins or occurrence of multiple discrete seizures with incomplete recovery of consciousness between episodes -Common causes: structural brain injury (e.g., stroke, tumor, head trauma), infection, metabolic abnormalities (e.g., hypoglycemia), medication noncompliance in patients with known epilepsy= condition is life-threatening (treatment needed) and is associated with systemic complications (e.g., cardiac arrhythmias, hypoventilation/hypoxia, disabling neurologic deficits) -IV lorazepam is a BZ that potentiates the effects of the inhibitory neurotransmitter GABA in the CNS (initial drug of choice for treatment of status epilepticus due to its efficacy and rapid onset of action); IV phenytoin (or fosphenytoin) is a long-acting anticonvulsant administered concurrently to prevent recurrence of seizure activity, regardless of patient responsiveness to lorazepam; phenytoin inhibits neuronal high-frequency firing in cortex by reducing ability of sodium channels to recover from inactivation option A= blocks T-type calcium channels in thalamic neurons; first-line treatment for absence seizures option C and D= BZs and barbiturates (e.g., phenobarbital) bind to ligand-gated chloride channel of GABA-A receptor; enhance inhibitor effect of GABA by increasing postsynaptic chloride influx option E= vigabatrin is an irreversible inhibitor of GABA transaminase, enzyme responsible for GABA catabolism in the CNS; can be used in cases of treatment-resistant epilepsy

64 years man -Episodic facial pain (2 months); sudden-onset, severe, electric shock-like-pain over right cheek and ear lasting for several seconds (shaving, washing face triggers it) -History: PTSD; exam: normal -Started on a single drug therapy; follow-up 3 months later (symptoms improved, but pancytopenia with 0.1% reticulocytes following drug therapy) -Mechanism of action of drug used? a)Blocks NMDA receptors in the hippocampal neurons b)Blocks serotonin reuptake on multiple levels c)Decreases calcium current in thalamic neurons d)Decreases sodium current on multiple levels e)Increases chloride current on multiple levels f)Inhibits synaptic acetylcholinesterase in cortical neurons

answer: Decreases sodium current on multiple levels Patient has trigeminal neuralgia (tic douloureux); presents with episodic, severe, unilateral, electric shock-like pain in distribution of CN V (triggered by chewing, washing face, shaving) First-line treatment= carbamazepine (inhibits neuronal high-frequency firing by reducing the ability of sodium channels to recover from inactivation) Carbamazepine may cause the following side effects: -Bone marrow suppression (pancytopenia; CBC should be monitored) -Syndrome of inappropriate antidiuretic hormone secretion leading to hyponatremia option A= ketamine, felbamate, and memantine are medications that block NMDA receptors; ketamine is used as general anesthetic, felbamate is an anticonvulsant, and memantine is used in Alzheimer option B= SSRIs (e.g., fluoxetine, sertraline) used to treat depression, anxiety and PTSD; side effects: headache, worsening anxiety/agitation, insomnia option C= ethosuximide blocks T-type Ca2+ channels and decreases calcium current in thalamic neurons; anticonvulsant of choice for absence seizures; side effects: GI symptoms and drowsiness option E= Benzodiazepines, barbiturates act on GABA-A receptor to increase chloride conductance option F= Donepezil; used to treat Alzheimer; side effects: cholinergic (GI symptoms, bradycardia, etc.)

8 years boy -Diagnosed with absence seizures (year ago and takes medication) -Doing well and has not had any seizures since medication -He eats balanced diet, 50th percentile for weight and height -Mechanism of drug? a)Binds to GABA receptors and increases the chloride current b)Binds to synaptic vesicles to modulate neurotransmitter release c)Decreases the calcium current in thalamic neurons d)Decreases the sodium current in cortical neurons e)Increases brain beta-hydroxybutyrate acid levels

answer: Decreases the calcium current in thalamic neurons -Ethosuximide is the first-line treatment for absence seizures -This drug blocks T-type Ca2+ channels that trigger and sustain rhythmical burst discharges in thalamic neurons -Side effects: minimal, nausea, vomiting, fatigue and hyperactivity -If not tolerated due to side effects, or drug is ineffective, valproic acid is used as second-line (inhibits voltage-dependent Na+ channels and also causes rise in GABA concentration) option A= BZs and barbiturates option B= levetiracetam binds to a synaptic vesicle protein (SV2A) to modulate release of GABA and glutamate option D= phenytoin, carbamazepine and valproic acid; reduce ability of sodium channels to recover from inactivation option E= ketone bodies; goal in ketogenic diet; this high-fat, low carb diet may benefit some patients with refractory epilepsy

64 years man -Slowing of his movements and gait instability (caused recent falls) -Diagnosed: parkinson (year ago); exam: resting tremor involving right hand and rigidity during passive flexion and extension of right upper limb -The patient is started on therapy with pramipexole; mechanism? a)Decreases central dopamine degradation b)Directly stimulates dopamine receptors c)Enhances the effect of endogenous dopamine d)Increases dopamine precursor availability in the brain e)Inhibits central muscarinic receptors

answer: Directly stimulates dopamine receptors Dopamine agonists have chemical structure similar to dopamine and directly stimulate dopamine receptors; don't have to be metabolized to be active; 2 classes of dopamine agonists: -Ergot compounds (derived from ergot fungi): bromocriptine -Nonergot compounds: pramipexole and ropinirole They are used in treatment of Parkinson as these medications have a long half-life and can delay need for levodopa, thereby postponing development of motor fluctuations until later in disease course; Bromocriptine also treated hyperprolactinemia option A= selegiline is used to delay progression of PD via inhibition of MAO-B in brain option C= amantadine; indirect and direct dopaminergic agent, it alleviates some of motor symptoms of PD by enhancing effects of endogenous dopamine, has some anticholinergic properties= reduces tremors option D= COMT inhibitors (e.g., entacapone, tolcapone) and carbidopa (dopa decarboxylase inhibitor) decrease peripheral breakdown of levodopa to dopamine= more goes to brain option E= e.g., trihexyphenidyl and benztropine); sometimes used to treat drug-induced parkinsonism and those with tremor as predominant symptoms of idiopathic PD

75 years man -Brought to ED after syncopal episode at assisted living facility (hour ago, while eating lunch, slumped over table, woke up after 30secs) -Appeared pale afterward, wasn't confused, blood glucose was normal -History: hypertension, T2D, benign prostatic hyperplasia, Alzheimer -Supine and standing BP= same; oropharyngeal mucosa is moist and JVP is normal; hospitalized= episodic bradyarrhythmia and AV block -Medication that caused this? a)Canagliflozin b)Diphenhydramine c)Donepezil d)Lisinopril e)Terazosin

answer: Donepezil -Brief loss of consciousness= syncope (due to transient loss of cerebral perfusion); most episodes are benign (vasovagal reflex= triggered by emotional stress or prolonged standing) -Syncope can be due to underlying pathology (e.g., cardiac arrhythmia, severe aortic stenosis, pulmonary embolism) or due to medications -Acetylcholinesterase inhibitors (e.g., donepezil, rivastigmine) are commonly used in management of Alzheimer dementia= improve cognitive function (dysfunction of cholinergic pathways in Alzheimer) -Mechanism also produces enhanced parasympathetic tone= age-related degeneration of conduction system is common in elderly= AChE inhibitors can precipitate bradycardia and AV block in such patients= leads to reduced CO that may manifest as presyncope or syncope -All other options= orthostatic hypotension

48 years woman -Lower extremity discomfort; no pain but describes uncomfortable sensation in her legs (particularly evening/night) -Relieved by leg movements or walking -Intermittently experiences mild migraines (recent episode= antiemetic therapy worsened leg discomfort) -No other abnormalities (labs, exam, etc.); pharmacotherapy has which of the following mechanisms of action? a)Beta-adrenergic antagonist b)Dopamine receptor agonist c)Muscarinic receptor antagonist d)Opioid mu-receptor agonist e)Selective serotonin reuptake-inhibitor

answer: Dopamine receptor agonist -Patient with vague lower extremity discomfort and urge to move legs that worsens at night= restless leg syndrome (RLS); RLS is characterized by uncomfortable sensations in legs that are often difficult to describe and associated with urge to move the lower extremities; symptoms are relieved with movement (e.g., walking, stretching) and are typically worse at night; as a result patients often develop sleep-onset insomnia -Pathophysiology= incompletely understood, CNS iron deficiency (even in patients with normal serum iron levels) and abnormalities in dopaminergic transmission -Certain medications are associated with worsening symptoms= dopamine antagonists (e.g., antipsychotics, metoclopramide), first-generation antihistamines (e.g., diphenhydramine) and antidepressants (e.g., SSRIs, mirtazapine)= choice E -Treatment: avoidance of aggravating factors, iron replacement, and dopamine agonists (e.g., ropinirole, pramipexole) which are thought to modulate CNS dopaminergic transmission and are rapidly effective in most cases option A= e.g., propranolol are used for treating akathisia (e.g., strong urge to move), which occurs most commonly as a side effect of dopamine antagonists, e.g., antipsychotics; akathisia is not accompanied by unpleasant sensation in legs and does not follow circadian rhythm option C= e.g., benztropine used in treatment of dystonia, characterized by sustained muscle contractions usually associated with abnormal posturing, e.g., torticollis option D= e.g., codeine are occasionally used for treating refractory RLS, but carry significant risks (e.g., sedation, abuse, respiratory depression)

22 years man -Change in behavior over past year; 9 months ago, patient abruptly decided to quit college in his last year and insisted on moving back home to "work on my research without being disturbed" -Since then, he has stopped going out with friends, seems uninterested in his usual hobbies of playing and watching football, and spends hours on his computer researching government cover ups -Stays up late at night, sleeps during the day, and showers only once a week; no motivation to look for work; he becomes agitated when mom encourages him to go back to school (accuses her of being one of the black suits); exam: poor hygiene -Medication? a)Alpha receptor agonism b)Dopamine receptor antagonism c)Dopamine reuptake inhibition d)GABA receptor agonism e)Histamine receptor antagonism f)Norepinephrine reuptake inhibition g)Serotonin reuptake inhibition

answer: Dopamine receptor antagonism -patient's paranoid delusions and negative symptoms (e.g., loss of motivation, social withdrawal, decreased attention to personal hygiene) lasting >6 months are consistent with schizophrenia, a psychotic disorder with typical onset in early adulthood; progressive functional impairment is common, as illustrated by this patient's academic and occupational dysfunction -Antipsychotic medications are first-line pharmacotherapy for schizophrenia; primary mechanism of action of antipsychotics is antagonism at postsynaptic dopamine D2 receptors; second-generation antipsychotics have the additional property of serotonin type 2 receptor antagonism, and some SGAs (e.g., aripiprazole) also act as D2 receptor partial agonists option A= mechanism of action of vasopressors, nasal decongestants option C and F= usually in combination with NE reuptake inhibition is the action of medications used in treatment of ADHD; e.g., methylphenidate, bupropion option D= ethanol, barbiturates, BZs and propofol option E= antihistamines; for allergy option G= SSRIs= antidepressants; although patients with schizophrenia often experience mood and anxiety symptoms, antidepressant medication would not effectively target psychotic symptoms

70 years man -Muscle rigidity (impairs daily activity); history: parkinson (5 years ago, levodopa/carbidopa) -Initial response= allowed him to continue usual activities; recently= episodes of sudden restricted mobility lasting for hours and sometimes "freezes" -Episodes alternate with symptom-free periods (mobile and feels well); patient desires relief -During discussion of patient's fluctuating mobility, which of the following should the physician emphasize? a)Drug holidays are helpful b)Drug response is unpredictable c)Fluctuations are caused by enhanced drug metabolism d)Fluctuations are temporary and will resolve spontaneously e)Stopping carbidopa is the best management

answer: Drug response is unpredictable -long-term treatment of Parkinson with levodopa can be complicated be periodic fluctuations in motor function; patients have good mobility during "on" periods and increased bradykinesia/rigidity during "off" periods ("on-off" phenomena); oftentimes, these fluctuations appear to correlate with serum drug levels (e.g., reduced mobility 4 hours after last dose); as PD progresses, the therapeutic window for levodopa narrows, possibly due to natural or levodopa-induced nigrostriatal degeneration -As a result, small changes in serum drug levels can result in motor fluctuations (e.g., mild elevations may result in dyskinesia, whereas reductions may result in bradykinesia/rigidity) -In advanced PD, motor fluctuations can occur independently of medication dosing and become unpredictable option A= effective in preventing motor fluctuations in PD; also, stopping levodopa abruptly is not recommended as it could cause a condition similar to neuroleptic malignant syndrome option C and D= progressive nigrostriatal degeneration, not enhanced drug metabolism is cause of motor fluctuations; as such, they are not temporary or self-limiting option E= stopping carbidopa (decarboxylase inhibitor) is inappropriate as carbidopa reduces peripheral conversion of levodopa to dopamine and consequently minimizes systemic side effects like nausea/vomiting

38 years woman -Breast tenderness and absence of menstrual periods over last 3 months -History: schizophrenia and familial hypercholesteremia -Medications: risperidone (improvement in psychotic symptoms) and atorvastatin; BMI down from 30 to 26 (6 months; strict diet) -Basic chemistry panel and thyroid function tests= normal; urine pregnancy test is negative -Explanation of patient's amenorrhea? a)21-hydroxylase deficiency b)Asherman syndrome c)Drug-induced amenorrhea d)Polycystic ovary syndrome e)Primary ovarian insufficiency f)Weight loss

answer: Drug-induced amenorrhea -Patient with amenorrhea and breast tenderness= drug-induced hyperprolactinemia due to risperidone, an antipsychotic drug used in schizophrenia; primary action is to inhibit dopamine D2 receptors, but it also has inhibitory effects on serotonergic and alpha-adrenergic pathways -Secretion of prolactin from the anterior pituitary is primarily regulated by inhibitory effect of hypothalamic dopamine, stimulation of D2 receptors on lactotrophs decreases synthesis and release of prolactin -Because risperidone and other antipsychotic drugs block D2 receptor activation, there is loss of normal tonic inhibition of prolactin release, leading to hyperprolactinemia in some cases= galactorrhea and breast soreness; in addition, elevated prolactin levels inhibit release of GnRH from the hypothalamus= central hypogonadism and amenorrhea option A= nonclassical 21-hydroxylase deficiency may present with menstrual irregularity (oligomenorrhea), hirsutism, and acne in women; presents in childhood or adolescence option B= presence of severe intrauterine adhesions that cause amenorrhea and infertility; occurs in patients who have had multiple dilation and curettage procedures or those with chronic endometritis option D= characterized by infertility, oligomenorrhea, and hyperandrogenism; begins in adolescence and is unlikely to develop in 38 years old woman; most patients are obese; insulin resistance is though to play in pathogenesis option E= characterized by amenorrhea, hypoestrogenism, elevated serum gonadotropin levels in women <40 years; decreased estrogenic effect on breast tissue can cause decline in breast density but wouldn't cause tenderness option F= e.g., anorexia nervosa, intense exercise, and/or severe stress can cause amenorrhea by inducing functional hypothalamic amenorrhea; but wouldn't cause breast tenderness

5 years boy -Recurrent, generalized tonic-clonic seizures over past 24 hours (high fever, flulike symptoms past 3 days) -History: febrile seizures at 6 months -Fever, lethargic and doesn't follow simple commands consistently -Neck is supple; during exam patient suddenly develops sustained, generalized tonic-clonic convulsions without fully regaining consciousness between episodes -Mechanism of action of initial therapy for his seizures? a)Blockade of presynaptic voltage-gated calcium channels b)Blockade of presynaptic voltage-gated sodium channels c)Enhanced postsynaptic chloride influx d)Inhibition of presynaptic neurotransmitter reuptake e)Inhibition of vesicle fusion and neurotransmitter release

answer: Enhanced postsynaptic chloride influx -Patient has status epilepticus, possibly due to complex febrile seizures or infectious meningitis/encephalitis; initial drug of choice for treating status epilepticus is an IV benzodiazepine (e.g., lorazepam), due its efficacy and rapid onset of action -BZs enhance effect of the inhibitory neurotransmitter GABA at the GABA-A receptor, a ligand-gated ion channel -Allosteric attachment of BZs to the GABA-A receptor causes increased influx of negatively charged chloride ions in response to GABA-binding= hyperpolarization of postsynaptic neuron and suppression of AP firing (anticonvulsant effect) option A and E= calcium influx= vesicle fusion and neurotransmitter release into synaptic cleft; Gabapentin is an anticonvulsant that works by blocking voltage-gated calcium channels, thereby attenuating excitatory neurotransmitter release option B= phenytoin; can be used to prevent recurrence of status epilepticus due to its long duration of action option D= SSRIs and SNRIs; used for depression and anxiety disorders

25 years woman -Newly diagnosed seizure, comes to ED with fever and skin rash -First seizure 6 weeks ago, MRI= no structural lesions -Started on phenytoin 4 weeks ago; fever and diffuse confluent erythema involving 60% of body, palpable generalized lymphadenopathy, symmetrical facial swelling -Lab findings present in this patient? a)Antineutrophil cytoplasmic antibodies b)Blasts with Auer rods c)Cryoglobulinemia d)Eosinophilia e)Microangiopathic hemolytic anemia

answer: Eosinophilia -patient's cutaneous findings and systemic findings after recently starting new medication= drug reaction with eosinophilia and systemic symptoms= DRESS syndrome -DRESS= life-threatening drug-reaction occurring 2-8 weeks after drug exposure; commonly associated drugs= anticonvulsants (e.g., phenytoin, carbamazepine), allopurinol, sulfonamides (e.g., sulfasalazine) and antibiotics (e.g., minocycline, vancomycin); mechanism unknown (drug-induced herpesvirus reactivation followed by clonal expansion of T cells that cross-react with the drug?) -Symptoms: fever, generalized lymphadenopathy, facial edema, diffuse morbilliform skin rash that can progress to a confluent erythema with follicular accentuation; liver (hepatomegaly, jaundice), kidney (acute interstitial nephritis), lung (cough, dyspnea) -Labs: eosinophilia, atypical lymphocytosis, elevated ALT and AST -Symptoms improve several weeks after drug withdrawal option A= drug-induced antineutrophil cytoplasmic antibodies-associated vasculitis is most commonly linked to medications for hyperthyroidism (e.g., propylthiouracil, methimazole) and hydralazine; patients can develop constitutional symptoms, arthralgias/arthritis, cutaneous vasculitis option B= acute myelogenous leukemia; complications of pancytopenia= fatigue, fever due to infection, ecchymosis or epistaxis option C= mixed cryoglobulinemia is a small to medium-vessel vasculitis caused by circulating Ig-complement complexes that precipitate on refrigeration; associated with chronic inflammatory states (e.g., SLE, hepatitis C) and can present with systemic findings (e.g., fatigue, arthralgias, myalgias) and palpable purpura in the lower extremities due to cutaneous vasculitis option E= usually presents with anemia and fragmented RBCs (schistocytes) on peripheral smear; associated with cancer chemotherapy drugs like cisplatin, cyclophosphamide

6 years boy -Difficulty with behavior in school (doesn't pay attention in class, grades are dropping, to the point he may need medication to stay focused) -At home he seems to stare into space for several seconds, multiple times throughout the day -Occurs even when watching his favorite TV shows, seems to ignore her when she calls his name; no previous history or medications -Older-brother= ADHD and aunt with epilepsy -Best treatment? a)Carbamazepine b)Ethosuximide c)Lorazepam d)Methylphenidate e)Phenobarbital

answer: Ethosuximide -Absence seizures, a type of generalized epilepsy, present in children age 4-10 as brief staring spells with momentary lapse in consciousness -Automatisms, such as rhythmic blinking or repetitive lip smacking, may occur; most absence seizures last approximately 10 seconds (behavior and awareness typically return immediately to normal following an attack) -Patient experiences no postictal period and usually has no recollection that a seizure has occurred; absence seizures are provoked by hyperventilation, and EEG= generalized 3-Hz spike wave complexes -Treatment: ethosuximide, which inhibits T-type calcium channels in thalamic neurons; Valproate can also be used if first-line therapy is not tolerated or ineffective; Childhood absence epilepsy is typically benign, and seizures often resolve before puberty option A and E= narrow-spectrum anticonvulsants (e.g., carbamazepine, gabapentin, phenobarbital, phenytoin) are generally favored for focal-onset seizures option C= BZ; first-line with status epilepticus option D= indirect acting sympathomimetic used to treat ADHD; although ADHD may present similarly with inattention, staring episodes are more consistent with absence seizures= can't be interrupted with verbal/tactile stimuli, because they don't occur due to boredom

32 years man -Brought to ED by police after an altercation at a local pub -Agitated and says that his chest hurts; high BP and pulse; exam: patient is combative and uncooperative -Pupils are dilated and reactive to light; exam: nasal perforation -ECG= myocardial ischemia; inhibition of which process is most likely for patient's presentation? (learn don't answer) a)A b)B c)C d)D e)E f)F g)G

answer: F -Patient is most likely acutely intoxicated with cocaine; cocaine is a stimulant that inhibits presynaptic reuptake of monoamines (NE, dopamine, serotonin), resulting in sympathetic stimulation (e.g., hypertension, tachycardia, light-responsive mydriasis) and CNS activation (e.g., increased arousal, euphoria, agitation, seizures) -Cocaine's ability to increase synaptic availability of NE, combined with some of its other effects (e.g., sodium channel antagonism), makes it a potent vasoconstrictor that can promote myocardial ischemia by causing coronary artery vasospasm and increased platelet aggregation (i.e., thrombus formation) -When cocaine is used intranasally, its vasoconstrictive effects cause local ischemia resulting in mucosal atrophy that can then lead to nasal septum perforation, these findings are highly suggestive of cocaine abuse

55 years man -Intermittent shortness of breath, cough, chest tightness (3 months) -History: childhood asthma (improved on adolescence, doesn't require inhalers or steroids since), he feels current symptoms are similar to prior asthma symptoms -Recently diagnosed with essential tremor (improved with medical therapy); exam: bilateral scattered expiratory wheezes -Symptom recurrence due to medication affecting? (learn don't answer) a)A b)B c)C d)D e)E f)F

answer: F -Patient developed symptoms of intermittent asthma (cough, wheeze, dyspnea) associated with new medication for essential tremor; ET is a common movement disorder, and beta blockers are often used as first-line therapy for debilitating or distressful symptoms; beta blockers prevent interaction of Epi and NE with receptors at adrenergic synapses; in ET= dampens sensitivity of muscle spindles, which improves tremor symptoms by smoothing out the force of contraction -Propranolol (most commonly used for ET)= nonselective beta blockers; in patients with reactive airway diseases (e.g., asthma, COPD), inhibition of beta-2 receptors can cause bronchoconstriction with resultant asthma symptoms (selective beta-1 blockers are better tolerated; but dose-dependent, at high concentrations can cause bronchoconstriction) option A= tyrosine is transported into cell by large neutral amino acid transporter option B= conversion of tyrosine to DOPA by tyrosine hydroxylase is the rate-limiting step in catecholamine biosynthesis; inhibited by drugs that were previously used to treat pheochromocytoma option C= reserpine inhibits vesicular monoamine transporter; thereby inhibiting dopamine entry into presynaptic vesicles and essentially causing a chemical sympathectomy; reserpine is occasionally used as an antipsychotic; it may cause wheezing in patients but is not indicated in treatment of ET option D= adrenergic neuron blockers such as guanethidine inhibit NE release; lowering BP and HR; occasionally used for chronic pain syndrome, doesn't cause bronchoconstriction but not used for ET option E= TCAs and cocaine work by inhibiting NE reuptake by presynaptic neurons; can worsen ET

45 years woman -History of opioid use disorder and HIV infection is brought to ED due to worsening lethargy -Receives methadone maintenance therapy, which she recently restarted after being hospitalized for an HIV-related infection -Infection improved after inpatient treatment, continues to take appropriate antimicrobials as an outpatient; low RR -Exam: somnolent and withdraws all extremities to painful stimuli; pupils are small and sluggish to react; brain imaging= now new findings and renal function tests are within normal limits -Use of which medication contributed to patient's current condition? a)Atovaquone b)Dapsone c)Fluconazole d)Rifampin e)Valganciclovir

answer: Fluconazole -Methadone is a mu-opioid receptor agonist used for maintenance treatment of opioid use disorder; it has a long half-life, which allows it to effectively suppress cravings and withdrawal symptoms; like all opioid medications, methadone carries a risk of overdose, which is characterized by somnolence, miosis, and respiratory depression (i.e., bradypnea); another side effect is QTc prolongation, which can predispose to torsade de pointes -Methadone is extensively metabolized by cytochrome P450, particularly by CYP3A4; in this case, the patient likely received fluconazole, an antifungal medication used in treatment of cryptococcal meningitis (which typically occurs in patients with a CD4 count <100 cell/uL) -Fluconazole inhibits CYP3A4, which can increase plasma methadone concentration and lead to opioid toxicity; interestingly, not all P450 3A4 inhibitors prolong methadone effects, other medications known to cause similar interactions with methadone: voriconazole, ketoconazole, ciprofloxacin, clarithromycin, cimetidine, and fluvoxamine option A and B= atovaquone and dapsone= alternative agents to prevent pneumocystis pneumonia in patients with trimethoprim-sulfamethoxazole allergy; side effects of dapsone= hemolytic anemia and methemoglobinemia, particularly in G6PD deficiency patients; neither have interactions with methadone option D= in combination with other antibiotics to treat multiple mycobacterial infections (e.g., TB, avium complex); P450 inducer= lower plasma concentrations of methadone option E= in treatment and secondary prophylaxis of CMV retinitis; side effects: bone marrow suppression and diarrhea

20 years student -Persistent fatigue, irregular menstrual periods, difficulty losing weight despite intensive exercise -Several times a week, episodes where she consumes large amounts of cookies, bread, potato chips (disgusted with herself afterward and does additional exercise) -Hates her appearance and constantly compares herself to her slimmer friends, but she denies feeling persistently depressed -Weight is 70kg, and height is 160cm; low BP and high pulse; exam: pharyngeal erythema and minimal parotid enlargement bilaterally -Low potassium level and high amylase; pregnancy test= negative; best medication? a)Bupropion b)Desipramine c)Fluoxetine d)Lisdexamfetamine e)No pharmacological treatment is effective f)Olanzapine

answer: Fluoxetine -Recurrent binge eating followed by compensatory exercise- accompanied by physical findings and lab values suggestive of self-induced vomiting (pharyngeal erythema, parotid enlargement, hyperamylasemia)= bulimia nervosa; other findings: tachycardia, hypotension, dry skin, menstrual irregularities, erosion of dental enamel, metabolic alkalosis -Treatment: nutritional rehabilitation, cognitive-behavioral therapy, pharmacotherapy with SSRI; fluoxetine is considered the drug of choice and has best evidence of being most effective in combination with nutritional and psychotherapy option A= NE-dopamine reuptake inhibitor antidepressant; contraindicated in bulimia nervosa due to elevated risk of seizures option B= TCA, not first-line in bulimia option D= used to treat binge eating disorder option F= antipsychotic associated with weight gain, used in patients with anorexia nervosa who fail to gain weight with psychotherapy alone

34 years woman -Sleep problems; difficulty falling asleep at night and exhausted during the day (year) -Family history: heart disease and depression -Cognitive-behavioral therapy for insomnia failed -She requests pharmacologic treatment and is prescribed short course of zolpidem; mechanism? a)Dopamine receptor antagonism b)GABA receptor agonism c)Histamine receptor antagonism d)Melatonin receptor agonism e)Orexin receptor antagonism f)Serotonin receptor agonism

answer: GABA receptor agonism -Zolpidem is short-acting nonbenzodiazepine hypnotic agent; other medication of this class (zaleplon and eszopiclone; Zzzz); like BZs, these medications bind to GABA-A receptor at BZ receptor site, where they as GABA-A agonists -They are chemically distinct from BZs, and their binding is more specific for certain receptor subtypes; as a result, nonbenzodiazepines are primarily hypnotics and do not produce the anxiolytic, muscle relaxant, or anticonvulsant effects associated with BZs -Nonbenzodiazepines cause similar side effects to BZs (e.g., next-day somnolence, cognitive dysfunction, increased risk of falls in older adults); they should be used for shortest time possible and lowest effective dose option A= antipsychotic medications (e.g., haloperidol, risperidone) option C= antihistamines (e.g., diphenhydramine); often used as over the counter sleep aids, little benefit in insomnia option D= ramelteon option E= suvorexant option F= triptans used to treat migraine headaches and psychedelics, e.g., psilocybin, LSD

31 years man -Hospitalized after sustaining multiple injuries during motor vehicle collision; not conscious initially, no information available about medical history -2 days after initial hospitalization= palpitations and difficulty sleeping; shortly after, experiences generalized tonic-clonic seizure -When he recovers= tells physician that he usually takes medications for anxiety, bipolar disorder, and chronic low back pain -Normal vitals; appears anxious, tremulous and diaphoretic; withdrawal from which of the following medications is most likely explanation of presentation? a)Lamotrigine b)Lithium c)Lorazepam d)Oxycodone e)Quetiapine f)Sertraline g)Valproate

answer: Lorazepam -patient's presentation is consistent with BZ withdrawal (e.g., lorazepam), which is most often characterized by rebound anxiety, tremor, insomnia, and sympathetic hyperactivity (e.g., diaphoresis, palpitations); in severe cases, psychotic symptoms (e.g., hallucinations, delusions), seizures, and even death may occur; risk of withdrawal is greatest with short-acting agents (e.g., alprazolam), prolonged used, and use of higher doses -BZs act on GABA-A= enhancing action of GABA; the body adapts to chronic BZ exposure by downregulating the GABA-A receptor, leading to tolerance, a phenomenon in which the same dose of a medication yields decreasing effects over time -The same adaptive downregulation process is responsible for the withdrawal syndrome when BZ is discontinued and the body is unable to balance opposing excitatory neurotransmitters; BZ withdrawal is treated by starting a patient on a long-acting BZ (e.g., diazepam) and planning for an extended, gradual taper option A and G= anticonvulsants lamotrigine and valproate are used in bipolar disorder for their mood-stabilizing properties; withdrawal from these medications would not produce seizures in a patient without underlying seizure disorder option B and E= withdrawal from lithium or quetiapine, when used to treat bipolar disorder, may be associated with risk for manic relapse and nonspecific rebound psychiatric symptoms (e.g., anxiety, restlessness); wouldn't explains patients symptoms or seizures option D= opioid withdrawal is characterized by nausea, vomiting, diarrhea, lacrimation, dilated pupils, and piloerection option F= SSRIs withdrawal presents with rebound anxiety/dysphoria, dizziness, headache, and nausea

62 years man -History: 10 years of T2D; over past 2 years= burning pain and pins and needle sensations in feet, which is worse at nighttime -Nortriptyline was prescribed and the dosage was gradually increased -Currently, takes maximal recommended dose, but states symptoms are not well controlled -No drug adverse effects; other medical history: hypertension, hyperlipidemia -Exam: decreased light touch and pain sensation in feet; ankle reflexes are decreased bilaterally; high HbA1c (7%); addition pharmacotherapy is planned -Medication that would provide additional benefit through a different mechanism with no drug-drug interaction? a)Amitriptyline b)Desipramine c)Duloxetine d)Gabapentin e)Venlafaxine

answer: Gabapentin First-line therapy for painful diabetic neuropathy includes the following 3 classes of medications: -TCAs (e.g., nortriptyline, amitriptyline, desipramine) modulate pain transmission by inhibiting voltage gated sodium channels in sensory nerves and increasing NE signaling in CNS -SNRIs (e.g., duloxetine, venlafaxine) alter central transmission of pain by increasing NE in central synapses -Anticonvulsants (e.g., gabapentin, pregabalin) inhibit the release of excitatory neurotransmitters by binding to voltage gated calcium modulators on nerve terminals This patient is being treated with nortriptyline, a TCA drug; his symptoms are uncontrolled; therefore, an additional medication with a different mechanism of action should be added to avoid additive or potentially even synergistic adverse effects Gabapentin doesn't inhibit serotonin and NE in CNS, less likely adverse effects with TCA

62 years man -Comes to ED due to right upper extremity weakness -30 minutes ago= developed difficulty holding a pen (grip was weak and couldn't lift his right arm), tried calling coworker but couldn't speak (never had these symptoms before) -No loss of consciousness, weakness of other extremities, vision abnormality, nor headaches -Symptoms resolved spontaneously by time paramedics arrived -History: hypertension, 30-pack year (smoker), high BP -Exam: no extremity weakness of sensory loss; left carotid bruit; non-contrast CT is normal; most appropriate pharmacotherapy? a)CNS-targeted immunomodulator b)GABA-A receptor modulator c)HMG-CoA reductase inhibitor d)Selective serotonin reuptake inhibitor e)Tissue plasminogen activator

answer: HMG-CoA reductase inhibitor -Patient with cardiovascular risk factors (i.e., hypertension, smoking) and a carotid bruit developed right upper-extremity weakness and aphasia that resolved without intervention= transient ischemic attack (TIA), which, in this case, likely occurred following atheroembolization or low flow from patient's carotid artery lesion -TIAs are defined as focal neurologic deficits that occur due to transient brain ischemia; tissue infarction doesn't occur, and brain imaging is normal; symptoms are typically localized to a singular vascular territory and resolve without intervention (typically <24 hours) -Patients with TIA are at increased risk of future stroke, therefore, intensive medical management is indicated; secondary prevention includes: antiplatelet agents (e.g., aspirin, clopidogrel) to prevent thrombus formation and statin therapy (HMG-CoA reductase inhibitor, e.g., atorvastatin, rosuvastatin) to reduce atherosclerotic plaque formation and lifestyle modifications option A= CNS-targeted immunomodulators (e.g., ocrelizumab) or glucocorticoids are used in treatment of multiple sclerosis; demyelinating disease that causes relapsing and remitting neurologic deficits; occurs in young women mainly and neurologic deficits are slowly progressive over course of days option B= BZs use for acute seizure termination; seizures can cause postictal paralysis but are typically preceded by stereotypes positive symptoms (e.g., repetitive limb movement, lip smacking), whereas TIA often cause negative symptoms (e.g., loss of limb function, aphasia); also, seizure patients have postictal confusion or sedation option D= for anxiety or depression option E= e.g., alteplase; used for thrombolysis in patients with acute, potentially debilitating, ischemic stroke, which would present with persistent neurologic deficits; TPAs are not indicated as patient's symptoms resolved

83 years woman -Sent to ED from nursing home for evaluation of mental status changes; at baseline, she has mild memory impairment but otherwise cognitively intact, calm, cooperative with nursing home staff -Over past 24 hours= increasingly combative, agitated and stayed up all night -Behavioral and environmental modifications have not been helpful; history: hypertension, history of anxiety and depression -Exam: patient unable to attend to conversation, mildly disoriented and cannot state the days of the week backwards; without provocation, she strikes out at a nurse's aide standing next to her; labs are normal except urinalysis (increased presence of WBCs and positive for nitrites) -In addition to antibiotic therapy, which medication is best for her behavioral symptoms? a)Clozapine b)Doxepin c)Haloperidol d)Lithium e)Lorazepam f)Temazepam

answer: Haloperidol -Delirium is an acute-onset "confusional state" characterized primarily by waxing and waning mental status changes and impaired attention -Disorientation, agitation, psychosis and sleep disturbances may also occur; delirium occurs secondary to an underlying medical condition, such as UTI, and therefore primary management is treating the underlying cause -The elderly and those with preexisting cognitive disorders are at higher risk for delirium and may present with varying degrees of agitation -High-potency, first generation antipsychotics (e.g., haloperidol) and some SGAs (e.g., quetiapine) can be used for acute treatment of agitation and psychosis associated with delirium; antipsychotic use is appropriate in treatment of delirium in elderly if the patient is at risk of acute harm to self or others and behavioral intervention have failed; under these conditions, the benefits of antipsychotics (i.e., the provision of safety) outweigh the potential risks when used at low doses and short durations option A= second generation antipsychotic that is reserved for patients with treatment-refractory schizophrenia; not used for short-term treatment of agitation in delirium due to risk of agranulocytosis option B= TCA with anticholinergic effects that can worsen delirium; used for treatment of insomnia in other settings option D= mood stabilizer; not used for short-term treatment of behavioral dyscontrol in nonmanic patients option E and F= BZs can worsen confusional states, particularly in the elderly; BZs are only appropriate for treatment of delirium due to alcohol or BZ withdrawal

-new inhaled agent (drug A) is tested in experiment designed to characterize its properties as a general anesthetic -Drug A's partial pressure in arterial blood is shown on graph below as a function of time after beginning inhalation -A similar curve for nitrous oxide is also shown; both drugs are administered at same partial pressure of inspired gas; which of the following best describes properties of new agent compared to nitrous oxide? (learn don't answer) a)Fast equilibration with the brain b)High blood/gas partitioning coefficient c)Low potency d)Low solubility in the blood e)Rapid onset of action

answer: High blood/gas partitioning coefficient -During anesthetic induction with a gaseous agent, the partial pressure of the anesthetic in the inspired gas; at this point, no more anesthetic can dissolve in blood and blood is said to be saturated with anesthetic; the speed at which the blood becomes saturated depends on solubility of gas in blood; highly soluble anesthetic dissolve easily in blood, and thus larger amounts must be absorbed before blood becomes saturated -Blood solubility of an anesthetic is indicated by its blood/gas partition coefficient- anesthetics with higher blood solubility have larger blood/gas partition coefficients; Nitrous oxide is poorly soluble gas with blood/gas partition coefficient of 0.47 (partial pressure rises quickly in blood and becomes saturated) -Drug A has higher solubility (i.e., larger blood/gas partition coefficient) as its partial pressure in blood rises more slowly (eventually saturated), by this point= far more drug A is dissolved in blood -Speed of anesthetic induction is determined by rate at which brain tissue take up agent; which is dependent on the solubility of anesthetic in blood; if an agent is poorly soluble= less gas needed to saturate blood and brain saturation occurs quickly (vice versa) option A, D, E= nitrous oxide option C= potency of a gas anesthetic is determined by minimal alveolar concentration; defined as concentration of gas in the lungs that produces the desired effect in 50% of patients; potent anesthetics require lower partial pressure to be effective

40 years women -Brought to ED due to ataxia and tremor; symptoms started gradually 2 weeks ago and worsened acutely over past 2 days -Having difficulty walking and almost fell while getting up from a chair -History: bipolar disorder and recently diagnosed hypertension -Medications: stable dose of lithium for many years and new BP medication that was started several weeks ago -Vital signs: normal limits; neurologic exam: resting tremor and difficulty with balance; drug interaction involving which medication is causing patient's condition? a)Amiloride b)Clonidine c)Hydralazine d)Hydrochlorothiazide e)Metoprolol

answer: Hydrochlorothiazide -Lithium has a very narrow therapeutic index; patients with acute toxicity initially develop symptoms of GI upset (e.g., vomiting, diarrhea) with later development of neurologic findings (e.g., neuromuscular excitability, delirium) as the drug penetrates the CNS -However, chronic lithium toxicity generally presents with gradual onset of neurologic symptoms, such as this patient's worsening ataxia and tremor -Lithium is similar to sodium in its properties and is almost exclusively excreted by the kidneys; like sodium, it is filtered and reabsorbed mostly in the proximal tubules (>60%); any cause of decreased glomerular filtration (e.g., volume depletion, congestive heart failure, cirrhosis) can increase proximal sodium/lithium reabsorption and lead to increased lithium retention -Thiazide diuretics limit sodium reabsorption in the distal tubule, causing mild volume depletion (and potential hyponatremia) that stimulates proximal tubular sodium/lithium reabsorption, leading to lithium toxicity over time; ACEIs and NSAIDs also impair lithium clearance and lead to toxicity option A= potassium sparing diuretic that works primarily on distal tubule and collecting duct by inhibiting sodium reabsorption; potassium-sparing diuretics often decrease serum lithium levels

66 years man -Diplopia (worsens toward end of day); no shortness of breath, trouble swallowing, nor generalized weakness -Currently: low-dose pyridostigmine (for his condition, good compliance, little side effects) -Exam: fatigable asymmetric ptosis and binocular horizontal diplopia -Infusion of edrophonium elicits significant improvement in symptoms; best next step in management? a)Add atropine b)Administer pralidoxime c)Change long-term therapy to edrophonium d)Increase pyridostigmine dosage e)Stop pyridostigmine temporarily

answer: Increase pyridostigmine dosage -Myasthenia gravis (autoimmune disease): caused by autoantibodies directed against nicotinic Ach receptors on postsynaptic membrane of neuromuscular junction; AP propagation is impaired as antibody binding leads to receptor degradation and complement-mediated damage of postsynaptic membrane -Typically present with fluctuating weakness that worsens over course of day and most often involves the extraocular (e.g., ptosis, diplopia) and bulbar (e.g., fatigable chewing, dysphagia) muscles -Long-acting AChE inhibitors (e.g., pyridostigmine) are used in symptomatic treatment of myasthenia gravis (increase Ach concentration in neuromuscular junction, prolonging its effect on postsynaptic membrane) -Exacerbation of myasthenia in patients being treated with AChE inhibitors may occur due to 2 reasons= 1) undertreated (receives suboptimal doses); not enough Ach in neuromuscular junction (myasthenic crisis); infusion of short-acting AChE inhibitor edrophonium (Tensilon test) increases neuromuscular transmission and can provide temporary improvement in symptoms (Choice C); Pyridostigmine dose should be increased in the long-term management of these patients -2) rare; cholinergic crisis; patient receives inappropriately high doses of AChE inhibitor and has excessive Ach in synaptic cleft= excessive stimulation of skeletal muscles and results in muscle that is refractory to future impulses (also presents with muscle weakness, however, because the neuromuscular junction becomes insensitive to Ach= edrophonium infusion produces no improvement in symptoms)= AChE inhibitors should be temporarily discontinued in these patients (Choice E) option A= muscarinic Ach receptor antagonist, may be used in organophosphate poisoning, and mydriatic agent (dilates pupils); doesn't affect nicotinic receptors option B= used in organophosphate poisoning; reverses effects of both muscarinic and nicotinic overstimulation by "regenerating" AChE in neuromuscular junction

25 years student -Anxiety evaluation; episodes of sudden-onset palpitations, dyspnea, diaphoresis (wants to run away before being seen) -Arrives early to classes to get seat near door so its easier to leave; she is worried to have another episode during exams next week -Doesn't drink or illicit drugs; diagnosed with panic disorder -Decides to pursue cognitive-behavioral therapy and start short-term anxiolytic medication that also has muscle-relaxant and anticonvulsant properties -Mechanism of medication? a)Antagonism at GABA-A benzodiazepine receptor sites b)Increased duration of chloride channel opening c)Increased frequency of chloride channel opening d)Inhibition of serotonin and norepinephrine reuptake e)Inhibition of serotonin reuptake

answer: Increased frequency of chloride channel opening -Benzodiazepines (e.g., clonazepam, lorazepam) bind and modulate GABA-A receptors on CNS neurons, resulting in an increased frequency of chloride channel opening in presence of GABA; this hyperpolarizes and stabilizes the membrane= less excitable= anxiolytic, muscle relaxant, and anticonvulsant properties -Due to risk of dependence, they are generally prescribed short-term in anxiety and alcohol withdrawal syndrome option A= Flumazenil; reversal of BZ overdose option B= barbiturates, e.g., phenobarbital, increase duration of opening of GABA-A receptor-chloride channel in presence of GABA; have anxiolytic properties, they are not indicated in anxiety disorders because of their potential to induce fatal respiratory depression option D= TCAs (e.g., amitriptyline) and SNRIs (e.g., venlafaxine); used to treat panic disorder, no muscle relaxant or anticonvulsant properties option E= SSRIs, e.g., fluoxetine; used for panic disorder, but no muscle relaxant or anticonvulsant properties

44 years woman -History: breast cancer; comes due to persistent back pain -Recently started radiation treatment for several metastases -Takes acetaminophen and NSAIDs but continues to have severe pain -Exam: tenderness over several lumbar vertebrae; neurologic exam: normal; started on oral morphine therapy; direct effect of medication on spinal cord neurons? a)Activation of sodium-calcium exchange mechanisms b)Blockage of voltage-dependent sodium influx c)Increased calcium influx into cells d)Increased chloride influx into cells e)Increased potassium efflux out of cells

answer: Increased potassium efflux out of cells -Morphine is an opiate analgesic that is commonly used to treat severe chronic pain in patients with cancer -There are 3 main opiate receptor subtypes (e.g., mu, delta, kappa) that mediate analgesia at spinal and supraspinal levels; all these are Gi coupled receptors; morphine and other commonly used opiates have high affinity for mu receptor, which exerts the strongest analgesic effect -in spine, opiates bind to mu receptors on the primary afferent neuron, resulting in closure of voltage-gated calcium channels, reduced calcium influx, and decreased excitatory neurotransmitter release from presynaptic terminal (choice C) -Opiates also bind to mu receptors on postsynaptic membrane, which opens potassium channels and leads to membrane hyperpolarization due to potassium efflux; opiate induced inhibition of synaptic activity in the CNS attenuates pain transmission, producing analgesia option A= digoxin; inhibits Na/K ATPase in myocardial cells= increase intracellular sodium= inhibition of sodium-calcium exchanger= increased intracellular calcium= increased cardiac contractility option B= local anesthetics (e.g., lidocaine) reduce pain transmission by blocking voltage-gated Na channels; usually for minor surgical procedures and spinal anesthesia option D= BZs used for anxiety, panic attacks, or insomnia

25 years man -Unresponsive (in ED); history: schizoaffective disorder and previous suicide attempts by prescription medication overdose -Recently started haloperidol and sertraline; high fever, high BP, high pulse -Exam: obtunded and diffusely rigid without clonus; labs: increased creatinine and creatinine kinase, leukocytosis -Antidote mechanism? a)Activation of presynaptic voltage-gated calcium channels of motor neurons b)Antagonism of peripheral and central serotonin receptors c)Blockade of central dopamine D2 receptors d)Inhibition of acetylcholinesterase at the neuromuscular junction e)Inhibition of calcium ion release from sarcoplasmic reticulum of skeletal muscle

answer: Inhibition of calcium ion release from sarcoplasmic reticulum of skeletal muscle -Patient has neuroleptic malignant syndrome likely caused by overdose of haloperidol; NMS is a life-threatening adverse reaction most commonly resulting from the use of antipsychotics (neuroleptic) medications, which block dopamine receptors (brain) -Manifestations: diffuse muscle rigidity, high fever, autonomic instability (hypertension, tachycardia), and altered sensorium -Labs: creatine kinase elevation (rhabdomyolysis), can result in acute renal failure -Treatment: discontinue causative agent and provide supportive care (e.g., IV hydration); in severe cases, dantrolene is administered= antagonizes ryanodine receptors and inhibits calcium release from SR; Bromocriptine, a dopamine agonist, has also shown clinical benefit in NMS option A= will trigger calcium influx in terminal bulb= increase release of Ach from terminal bulb onto muscle endplate, and therefore, promote skeletal muscle contraction option B= serotonin receptor antagonists (e.g., cyproheptadine) can be used to treat serotonin syndrome; similar to NMS, serotonin syndrome can present with altered mental status and autonomic instability (diaphoresis, hypertension, hyperthermia, tachycardia); but it presents with hyperreflexia and myoclonus (not diffuse rigidity) and GI symptoms (nausea, diarrhea, vomiting) option C= leads to extrapyramidal symptoms (e.g., diffuse rigidity) like in NMS option D= results in increased Ach at neuromuscular junction, promoting skeletal muscle activation and contraction; should be avoided in NMS, they exacerbate conditions

-Newly developed inhaled anesthetic has improved safety profile compared to traditional fluorinated anesthetics -New agent demonstrates relatively high potency; which parameter best correlates with potency of inhaled anesthetic? a)Arteriovenous concentration gradient b)Blood/gas partition coefficient c)Hepatic metabolism rate d)Minimal alveolar concentration e)Steepness of arterial tension curve

answer: Minimal alveolar concentration -Potency of inhaled anesthetic is determined by the minimum concentration in the brain necessary to achieve an adequate level of anesthesia -When inhaled anesthetic is administered at a constant rate over a prolonged period of time, its partial pressure in the brain equals its partial pressure in other compartments (inhaled air, lungs, and arterial blood); since an anesthetic's concentration in the brain tissue is directly proportional to its partial pressure, it is possible to use the steady state alveolar partial pressure of the inhaled agent as a measure of anesthetic potency -The minimum alveolar concentration (MAC) refers to the percentage of anesthetic in the inspired gas mixture that renders 50% of patients unresponsive to painful stimuli -It corresponds to ED50 on dose-effect curve; potency is inversely proportional to the MAC, the lower the MAC the more potent the anesthetic -MAC is an intrinsic property of the anesthetic that does not depend on the type of the surgery, duration of anesthesia, or sex, height, etc.; it does, however, depend on body temperature and also decreases with increasing patient's age

20 years student -Falling asleep at inappropriate times; irresistible urge to sleep during day, these "naps" occur unpredictably (at start of class or during exam); sleeps 6-7 hours per night and feels refreshed in morning -Also experiences frightening auditory hallucinations before sleep; recently involved in motor vehicle collision (fell asleep at wheel) -Best treatment? a)Carbamazepine b)Clonazepam c)Melatonin d)Modafinil e)Zolpidem

answer: Modafinil -Narcolepsy is characterized by episodes of irresistible sleep during the day and usually one or more REM sleep-related phenomena such as cataplexy (i.e., sudden weakness triggered by strong emotions), hypnagogic/hypnopompic hallucinations (i.e., hallucinations while falling asleep or awakening, respectively); etiology: low levels of stimulatory neurotransmitter orexin (i.e., hypocretin)= involved in maintaining wakefulness and suppressing REM sleep-related phenomena -Treatment: modafinil (nonamphetamine stimulant), first-line, effective and well-tolerated and its misuse is rare -Modafinil= enhances dopaminergic signaling -Amphetamines (second-line agents)= sympathomimetic side effects (e.g., hypertension, arrhythmia, psychosis) and risk for misuse and dependency option A= used to treat seizures, bipolar disorder, trigeminal neuralgia option B and E= BZ; and nonbenzodiazepine (zolpidem)= used for insomnia; contraindicated in this case option C= used for treating jet lag and delayed sleep-wake phase syndrome

34 years man -Brought to ED due to severe headache, blurry vision (suddenly after having lunch at a new Italian deli) -Ate sandwich with lots of fancy meats and cheeses and drank iced tea -History: treatment-resistant major depressive disorder -BP is high (hypertensive emergency), HR is high; Exam: tremulous and diaphoretic -Medication used for his depression affects which step? a)Binding to postsynaptic monoamine receptors b)Monoamine breakdown c)Presynaptic nonselective monoamine uptake d)Presynaptic selective dopamine uptake e)Presynaptic selective norepinephrine uptake f)Presynaptic selective serotonin uptake

answer: Monoamine breakdown -Patient is experiencing a hypertensive emergency (e.g., severe hypertension, headache, blurry vision) and has signs of excessive sympathetic activity (e.g., tachycardia, diaphoresis, tremor), most likely due to food-drug interaction between tyramine-containing foods (e.g., aged cheeses, cured meats, draft beer) and MAOI -MAOIs increase the availability of all 3 major monoamines (e.g., dopamine, NE, serotonin) by inhibiting MAO; a mitochondrial enzyme that normally breaks down monoamines -Tyramine is an indirect sympathomimetic found in certain foods (e.g., aged cheeses, cured meats, draft beer) that is usually broken down in GI by MAO; In MAOI treated patients, tyramine escapes degradation and enters systemic circulation, leading to hypertensive crisis -Due to risks of tyramine-induced hypertensive crisis and serotonin syndrome, MAOIs are typically reserved for treatment-resistant depression option C= TCAs and bupropion= latter used for depression and tobacco dependence option E= SNRIs; e.g., atomoxetine used in ADHD option F= SSRIs

24 years man -Found unconscious in basement; history of substance abuse and recently hospitalized for major depressive disorder -In ED= cyanotic, unresponsive to painful stimuli; low BP and RR (4/min); pupils are constricted and minimally responsive to light -Resuscitation + IV medication and regains consciousness 2 mins later -Drug used to treat patient has greatest affinity to which receptors? a)5-HT b)Delta c)GABA-A d)GABA-B e)Kappa f)Mu g)NMDA

answer: Mu -History of substance abuse, miosis, profound sedation and respiratory depression with rapid reversal of signs/symptoms with antidotal therapy= opioid overdose -Several opioid receptor subtypes, including mu, delta, and kappa identified; available opioid medications (e.g., morphine, hydromorphone) produce therapeutic analgesic effects by binding to these receptors and modulating pain perception -Naloxone is a pure opioid receptor antagonist devoid of any agonist properties; it antagonizes opioid effects at all receptor types, it has greatest affinity to mu receptors (responsible for opioid-induced cardiorespiratory depression); naloxone competes with opioid medications, displacing them from opioid receptors; it is therefore an ideal agent for reversing opioid intoxication/overdose -Reversal effects can occur within mins, and duration is dose dependent (usually 1-4 hours); must be given parenterally because it is metabolically inactivated by liver when administered orally option A= Cyproheptadine, a 5-HT1A and 5-HT2A antagonist= used to treat serotonin syndrome; Ondansetron is a 5-HT3 antagonist= IV for nausea and vomiting option B and E= modulate pain; naloxone binds them, but with less affinity option C= BZs are agonists; flumazenil is an antagonist here for BZ overdose option D= baclofen is an agonist here; skeletal muscle relaxant for cerebral palsy option G= ketamine is an NMDA antagonist; general anesthetic

46 years woman -Follow-up after episode of alcoholic pancreatitis -Hospitalized 4 weeks ago for abdominal pain and concurrently underwent treatment with benzodiazepines for alcohol withdrawal -Prior to hospitalization, the patient was drinking 3L of boxed wine daily, however, she has remained abstinent since discharge -She was interested in medication for her alcohol use disorder, but lives alone and is worried that her cravings will overwhelm her self control; medication with which mechanism is most effective? a)Aldehyde dehydrogenase inhibition b)Antagonism at postsynaptic dopamine receptors c)Mu-opioid receptor blockade d)Partial agonism at nicotinic acetylcholine receptors e)Serotonin reuptake inhibition

answer: Mu-opioid receptor blockade -In addition to psychosocial interventions (e.g., motivational interviewing, alcoholic anonymous), several medications are available to treat alcohol use disorder, including naltrexone and acamprosate -Naltrexone blocks the mu-opioid receptor and inhibits the rewarding and reinforcing effects of alcohol, helping to reduce cravings and improving motivation to quit -Long-acting depot form (given as monthly injections) is useful for patients at risk for nonadherence with daily administration -Acamprosate, another abstinence promoting drug, works by modulating glutamate neurotransmission at the NMDA receptor option A= disulfiram, considered second-line treatment for alcohol use disorder because of poorer efficacy (effectiveness depends on supervised administration); when alcohol is ingested by a patient taking disulfiram, acetaldehyde accumulates and causes unpleasant adverse effects like nausea, vomiting, flushing, sweating and headache option B= antipsychotic medications option D= varenicline acts this way and prevents the nicotine stimulation of mesolimbic dopamine system associated with nicotine addiction option E= SSRIs= antidepressants, some efficacy in reducing alcohol use in patients with alcohol use disorder who have comorbid depression

60 years farmer -Acute-onset confusion, muscle weakness, and difficulty breathing -Vomited twice and has had 3 loose bowel movements over past 2 hours; history: insignificant -Pupils are constricted, symmetric, reactive to light -Exam: excessive sweating, lacrimation, and wheezing, incontinent of urine; IV atropine is administered -Which will persist even with treatment? a)Bradycardia b)Bronchospasm c)Diarrhea d)Miosis e)Muscle weakness f)Urinary incontinence

answer: Muscle weakness -Exposed to organophosphate, a cholinesterase inhibitor that is commonly used in agricultural pesticides; they inhibit cholinesterase in both muscarinic and nicotinic cholinergic synapses= decreased Ach degradation; Overabundance of Ach= overstimulation of receptors -Widespread muscarinic activation symptoms (diarrhea, urination, miosis, bronchospasm, emesis, lacrimation, salivation); nicotinic activation leads to muscle fasciculations, weakness, and paralysis; activation of both receptors in the CNS leads to seizures and coma -Atropine= competitive inhibitor of Ach at muscarinic receptors, thereby reversing muscarinic symptoms only, no effect on nicotinic receptors (e.g., still at risk of muscle paralysis) -Pralidoxime is a cholinesterase-reactivating agent that allows for degradation of excess Ach and treats both muscarinic and nicotinic effects of organophosphates

24 years man -Hospitalized after violent episode in which he destroyed his TV with baseball bat -Over past year, patient convinced that people on TV are monitoring his thoughts and movements and that certain newscasters were laughing at him -At night, unable to sleep and reported hearing voices of demons threatening to kill him -Diagnosed with schizophrenia and initially treated with chlorpromazine; now reluctant to take this medication, saying, "I don't like how it makes me feel"; his medication is subsequently switched to fluphenazine; compared to patient's initial treatment, fluphenazine is more likely to cause which of the following? a)Constipation b)Dizziness c)Dry mouth d)Hyperglycemia e)Muscular rigidity f)Sedation g)Urinary retention

answer: Muscular rigidity -FGAs, also known as typical antipsychotics, effective in treating schizophrenia and psychosis due to other causes -FGAs are classified according to potency, which refers to dose required to produce an effect -Low-potency FGAs primarily cause histaminergic, cholinergic, and noradrenergic blockade side effects; highly sedating, and often cause anticholinergic side effects and orthostatic hypotension -High potency FGAs are associated primarily with extrapyramidal symptoms (e.g., dystonia, akathisia, parkinsonism) due to potent D2 antagonism in nigrostriatal pathway -Patients medication changed from low potency to high potency FGA= muscular rigidity (drug-induced parkinsonism) is more likely to occur option A, C, and G= anticholinergic side effect with low potency FGAs option B= orthostatic hypotension due to alpha-1 blockade with low potency FGAs option D= metabolic side effect with SGAs option F= histamine receptor blockade with low potency SGAs

25 years man -Severe agitation and aggressive behavior; speaks loudly and rapidly, starting that he has "superpower" of mind control and that violence is the only way to defend against "the conspiracy" -History: bipolar disorder (olanzapine, lithium, fluoxetine; compliant) -Several doses of medication in ED to calm down, and admitted -Following night= lying very still on bed, doesn't respond to questions -High fever, high BP and pulse; diaphoresis and diffuse rigidity in upper and lower extremities -Explanation for current symptoms? a)Acute dystonic reaction b)Akathisia c)Drug-induced parkinsonism d)Lithium toxicity e)Neuroleptic malignant syndrome f)Serotonin syndrome

answer: Neuroleptic malignant syndrome -Patient is experiencing manic episode with psychotic features, most likely received multiple doses of antipsychotic medication to control his agitation in ED -His hyperthermia, sympathetic hyperactivity (e.g., diaphoresis, tachycardia) and severe muscle rigidity (lead-pipe rigidity), and altered mental status= neuroleptic malignant syndrome -NMS is life-threatening adverse reaction to antipsychotics, that typically presents 1-3 days after initiation or dose escalation -Caused by antagonism of central dopaminergic systems involved in thermoregulation and regulation of muscle tone and movement -Treatment: cessation of causative agent; BZs and dantrolene (a postsynaptic muscle relaxant) are used for treating severe cases option A= due to antipsychotics; distressing contractions of neck, mouth, and tongue option B= after antipsychotics; restlessness and inability to sit still option C= due to antipsychotics; rigidity, bradykinesia, and tremor option D= begins with nausea, vomiting, diarrhea, with neuromuscular signs developing later option E= same symptoms but would present with clonus and hyperreflexia

-Studying therapeutic options for treatment of neuromuscular disorders -New drug that seems to have partial agonist/antagonist activity against receptor X -When drug applied to cells w/ receptor X= immediate change in transmembrane calcium sodium, and potassium flow secondary to opening of receptor channel -Receptor X is most likely? a)Alpha-1 adrenoreceptor b)Beta-1 adrenoreceptor c)Beta-2 adrenoreceptor d)Muscarinic cholinergic receptor e)Nicotinic cholinergic receptor

answer: Nicotinic cholinergic receptor -Nicotinic receptors of the neuromuscular junction are an example of ligand-gated ion channel receptor (inotropic receptor) -AP through presynapse= calcium influx= fusion of vesicles= release of Ach= interact with nicotinic receptors on motor endplate= ion channel opens (receptor)= sodium and calcium influx and potassium efflux= generation of endplate potential -Nicotinic receptors also found in CNS on postganglionic neurons of autonomic ganglia -All other options= metabotropic receptors (use second messengers)

30 years woman -Chronic anxiety and tension; worries constantly about multiple issue involving work, finances and family -At work, difficulty concentrating as she repeatedly calls to check on whereabouts and safety of her husband and children -Describes frequent tension headaches, back pain and fatigue -Doesn't feel depressed and says that her sleep and appetite are normal; labs and thyroid tests are normal; in addition to psychotherapy, pharmacotherapy with buspirone is initiated -Medication associated with which properties? a)Antiseizure properties b)Efficacy in panic disorder c)Euphoric effects d)Muscle relaxant properties e)No risk of dependence f)Rapid onset of action

answer: No risk of dependence -Chronic anxiety and associated symptoms of muscle tension, impaired concentration, and fatigue are consistent with a diagnosis of generalized anxiety disorder -Buspirone is a nonbenzodiazepine anxiolytic used in treatment of GAD that has no risk of dependence in contrast to substantial risk associated with BZs -Buspirone acts as a partial agonist of the 5HT1A receptor; affects serotonin and has no muscle relaxant or anticonvulsant properties (BZs do); also it is not effective in treating panic disorder -Buspirone has lower sedative and hypnotic effects= doesn't cause euphoria; and requires up to 2 weeks for anxiolytic effect to begin

44 years man -Paresthesia, pain (feet); tingling and numbness in toes (several months ago, which gradually progressed to rest of both feet); burning pain on soles (disrupts sleep) -History: HIV (6 years; not treated initially but now takes antiretrovirals regularly); CD4 count (550/mm3) -Foot exam: decreased pain and touch sensation and loss of ankle reflexes bilaterally -Medication review: no neurotoxic drugs and labs are normal -Treatment with duloxetine is planned; mechanism? a)Decreased sensitization of sensory afferent neurons b)Decreased substance P-induced activation of dorsal horn neurons c)Frequency-dependent blockade of neuronal conductance d)Inhibition of GABA release by periaqueductal gray neurons e)Norepinephrine-mediated descending pain pathway modulation

answer: Norepinephrine-mediated descending pain pathway modulation This patient with HIV peripheral neuropathy is being treated with duloxetine (SNRI); SNRIs increase NE in central synapses, so they modulate descending pain pathway, leading to decreased central perception of pain First-line therapy for painful neuropathy includes the following 3 classes of medication -TCAs (e.g., nortriptyline, amitriptyline) modulate pain transmission by inhibiting voltage-gated sodium channels in sensory nerves and increasing NE signaling in the CNS -SNRIs (e.g., duloxetine, venlafaxine) after central transmission of pain by increasing NE in central synapses -Anticonvulsants (e.g., gabapentin, pregabalin) inhibit release of excitatory neurotransmitters by binding to voltage-gated calcium modulators on nerve terminals option A= NSAIDs; effective in pain due to inflammation but not for neuropathic pain option B and D= Opioids reduce pain perception at multiple levels; at spinal-level, they lead to decreased substance P-induced activation of dorsal horn neurons; at the supraspinal level, they inhibit GABA release by the periaqueductal gray neurons; effective for neuropathic pain, but not recommended as initial treatment due to risk of abuse and dependence option C= Carbamazepine, used for trigeminal neuralgia

43 years man -Multiple medical problems; seizure disorder, mental illness comes to office for first time to establish primary care physician -Routine physical exam= gingival hyperplasia and lost one incisor tooth -Cause? a)Phenytoin b)Lithium c)Prednisone d)Cocaine e)Methimazole f)Clozapine

answer: Phenytoin -Gingival hyperplasia, common side effect of phenytoin (50% in 3-4 months therapy); due to increased expression of PDGF -When gingival macrophages are exposed to increased amounts of PDGF, they stimulate proliferation of gingival cells and alveolar bone (may regress after discontinuation of phenytoin) -Phenytoin is an anticonvulsant effective in treatment of grand mal (tonic-clonic) seizures, partial seizures, and status epilepticus; phenytoin inhibits abnormal electrical activity of the brain by blocking voltage-gated sodium channels in neurons -Phenytoin has a narrow therapeutic index with a number of side effects: 1) mainly CNS manifestations; cerebellum and vestibular system (ataxia, nystagmus) 2) long-term therapy= gingival hyperplasia, coarsening of facial features, and hirsutism 3) phenytoin interferes with metabolism of folic acid (megaloblastic anemia) 4) induces P450; increases metabolism and decreases blood levels of many medications 5) during pregnancy= fetal hydantoin syndrome option B= narrow therapeutic index; toxicity associated with neuromuscular excitability (irregular coarse tremors, fascicular twitching, rigid motor agitation, ataxia); can cause nephrogenic diabetes insipidus, hypothyroidism, and cardiac conduction defects option C= hyperglycemia, weight gain, depression, fluid retention with edema, osteoporosis option D= blocks reuptake of NE, serotonin, dopamine in CNS; can cause cardiac ischemia, hypertension, seizures; cocaine withdrawal is associated with severe depression option E= inhibits thyroid hormone synthesis by suppressing iodination and coupling of tyrosine; side effects= edema, rash, agranulocytosis option F= atypical antipsychotic that is most helpful in treating negative symptoms of schizophrenia; side effects: agranulocytosis and seizures

42 years man -Enlarging mole (forearm, several years, rapidly increasing in size past 4 months); exam: 8-mm raised lesion with variegated pigmentation and ill-defined border on right forearm -Excisional biopsy is planned; local anesthesia is administered via an infiltrating injection of lidocaine mixed with epinephrine -Why use epinephrine? a)Altered neurotransmitter levels of nociceptive fibers b)Decreased pain during subcutaneous infiltration c)Increased anesthesia efficacy by synergistic effect d)Increased anesthesia potency by decreasing tissue pH e)Prolonged duration of action of the anesthetic agent

answer: Prolonged duration of action of the anesthetic agent Lidocaine is a local anesthetic that blocks transmission from free nerve endings. It is often mixed with epinephrine, which causes profound vasoconstriction when injected subcutaneously; this results in: -A decrease in local bleeding, making the procedure easier to complete -A prolonged duration of action because more lidocaine remains at site of injection -A decrease in systemic absorption, reducing potential for serious adverse effects (e.g., arrhythmias, seizures) option A= although Epi acts as a catecholamine neurotransmitter in the CNS, it mediates autonomic function in PNS; glutamate and substance P are the main neurotransmitters in pain sensation in the PNS option B and D= local anesthetics are weak bases that exist in charged and uncharged forms; the uncharged form must diffuse through the cell membrane and bind the inside portion of sodium channels in order to prevent depolarization; increased acidity in the surrounding tissue results in more anesthetic molecules remaining in the charged form, preventing them from penetrating the cell membrane to exert an effect; therefore, sodium bicarbonate is often added to lidocaine with epinephrine to increase the pH, which improves potency; increase in pH also decreases pain at injection site option C= produce same result by different mechanisms (e.g., opioids + acetaminophen)

52 years woman -Hand tremor (several months); prominent while carrying out simple daily activities such as drinking from a glass or pouring from a tea kettle -Tremor subsides somewhat when she drinks small amounts of alcohol -Family history: mother has same problems -Exam: bilateral tremor in upper extremities; best treatment? a)Benztropine b)Clozapine c)Diazepam d)Haloperidol e)L-DOPA f)Propranolol

answer: Propranolol -Has essential tremor; referred to as familial tremor as it often follows autosomal dominant inheritance; positive family history of tremor can sometimes assist in making diagnosis -Worsens while maintaining particular posture (e.g., holding object); improves with alcohol consumption -Propranolol= first-line treatment option A= anticholinergic agent used to treat tremor in parkinson disease and counteract extrapyramidal side effects of neuroleptic medications such as haloperidol option B= atypical antipsychotic that blocks D2 and 5HT2 receptors in CNS option C= BZ effective in seizures and alcohol withdrawal option D= used to treat positive symptoms of psychosis, e.g., delusions, hallucinations option E= used in combination with carbidopa to treat Parkinson; parkinson has resting tremor

76 years man -Severe insomnia (several months); trouble falling asleep, feels tired and irritable during the day -History: hypertension, prior cerebrovascular accident with residual right leg weakness -Medications: aspirin, rosuvastatin, lisinopril (compliant) -In addition to nonpharmacological interventions for insomnia, short-term pharmacotherapy is being considered -Medication with lowest risk of causing adverse effects in this patient? a)Alprazolam b)Amitriptyline c)Clonazepam d)Diazepam e)Diphenhydramine f)Quetiapine g)Ramelteon

answer: Ramelteon -Insomnia is common in elderly; sleep hygiene measures, cognitive-behavioral therapy (recommended as initial management) -When they fail, pharmacotherapy may be indicated but must be used with extreme caution in elderly and should be limited to short-term management (increased susceptibility to adverse effects in general and sedative-hypnotic agents in particular, e.g., oversedation, dizziness, confusion, impaired psychomotor performance, ataxia) -Melatonin agonist ramelteon has demonstrated the lowest side-effect burden in older adults; binds with high affinity to melatonin receptors in suprachiasmatic nucleus option A, C, and D= BZs should be avoided= cognitive impairment risk, delirium, falls, fractures and motor vehicle accidents option B and E= TCAs and diphenhydramine= strong anticholinergic effects, risk of sedation, delirium and falls option F= second-generation antipsychotic that would not be appropriate as first-line treatment for insomnia; antipsychotics carry risk of increased falls, cardiac mortality

23 years woman -Brought to ER after having tonic-clonic seizure -History: seizures (phenytoin); exam: no focal abnormalities -Low phenytoin plasma level (although compliant with medication); which of the following drugs may be responsible for patient's condition if co-administered with phenytoin? a)Amoxicillin b)Rifampin c)Metronidazole d)Fluconazole e)Cimetidine f)Omeprazole g)Oral contraceptives

answer: Rifampin Phenytoin is metabolized by hepatic P450 oxidase; features of phenytoin metabolism: -Hepatic hydroxylation of phenytoin is dose-dependent; low doses= sufficient number of enzyme molecules are available and drug is eliminated rapidly; higher doses= fully saturate the enzyme, hence, increasing dose above saturation limit causes inability to metabolize the drug and a rapid rise in plasma levels; even small increase in phenytoin intake above prescribed dose may lead to severe toxicity -Phenytoin is an inducer of P450 oxidase; induction of this enzyme increases consumption of many medications that are metabolized by the liver (such as oral contraceptives)= reduces medication concentration -Levels of phenytoin affected by medications that induce or inhibit P450; co-administration of phenytoin with P450 inducers decreases concentration of phenytoin in plasma and diminishes its effects (P450 inducers: barbiturates, rifampin, carbamazepine, griseofulvin, and chronic alcohol consumption; P450 inhibitors are isoniazid, cimetidine, omeprazole, azole antifungals, and grapefruit juice)

28 years man -Undergoing laparoscopic appendectomy for acute appendicitis -Shortly after induction of anesthesia, the surgery reports difficulty maintaining abdominal insufflation due to severe muscle rigidity -High pulse, ECG= sinus tachycardia; exam: diffuse muscle stiffness; inhibition of which of the following most likely will improve symptoms? a)Acetylcholine degradation in cholinergic synapses b)Alpha-receptor-mediated sympathetic activity c)GABA-mediated neuron hyperpolarization d)Ligand-receptor interaction in cholinergic synapses e)Sarcoplasmic calcium release in skeletal muscles

answer: Sarcoplasmic calcium release in skeletal muscles -During laparoscopic procedures, small incisions are created to insert instruments into the abdominal cavity; gas is insufflated to allow better visualization and easier access to organs while operating; difficulty maintaining abdominal insufflation due to muscle contraction is often caused by insufficient doses of paralytic agents; however, if the patient has been appropriately medicated, sustained muscle contraction, especially when widespread, may be due to malignant hyperthermia -Cause: abnormal ryanodine receptors (RYR1) in skeletal muscle; on exposure to certain anesthetic agents (typically succinylcholine and inhalational anesthetics)= abnormal receptors allow the unregulated passage of calcium from the SR into the intracellular space -As a result of excessive calcium accumulation, patients develop sudden-onset, widespread muscle rigidity -Sustained muscle contraction produces excessive CO2, leading to hypercarbia and acidosis, depletes cellular oxygen, leading to lactic acid production and causes muscle breakdown, leading to rhabdomyolysis; hyperthermia is often a later clinical sign that develops as increased cellular metabolism overwhelms the body's ability to dissipate heat -Muscle relaxants work upstream from the pathologic process of excessive calcium release and are therefore ineffective during an acute MH crisis; Dantrolene is used to treat MH because it directly inhibits intracellular calcium release from the abnormal RYR1 option A= neostigmine is an AChE inhibitor that prevents degradation of Ach in neuromuscular junction; it could increase muscle contractions option B= alpha antagonists (e.g., phentolamine, phenoxybenzamine) inhibit alpha-receptor-mediated sympathetic activity; they are used to lower BP by decreasing vascular tone; minimal activity on muscles Option C= baclofen is a GABA-B agonist that increases neuron hyperpolarization in CNS to relax muscles; works upstream of defect option D= nondepolarizing neuromuscular blocking agents (e.g., rocuronium) are nicotinic receptor antagonists that inhibit the action of Ach at motor endplate; works upstream of defect

22 years student -Seeks treatment for depressed mood, low energy, poor concentration, feelings of worthlessness and guilt following a breakup with her partner of 2 years -Diagnosed: major depression and treated with bupropion and psychotherapy -After 3 weeks of treatment; energy and motivation have improved somewhat; her grades in college are stabilizing (continue to receive psychotherapy) -However, she still feels "really down" at times and has poor appetite; further history suggests that the patient consumes very little food each day (pattern that preceded her use of bupropion) and exercises excessively -Due to her ongoing mood problems, increasing dose of antidepressant is considered; which of the following is a potential side effect of increasing dose? a)Agranulocytosis b)Hypertension c)Seizures d)Sexual dysfunction e)Stevens-Johnson syndrome f)Weight gain

answer: Seizures -Bupropion is an antidepressant that works by inhibiting reuptake of NE and dopamine; commonly used to treat major depression and is often preferred by patients because it does not cause weight gain or sexual side effects -Seizures are potential side effect of bupropion therapy, especially when the medication is given at high doses; the risk is even greater with pre-existing seizure disorders with potential electrolyte imbalances -Bupropion is therefore contraindicated in patients with seizure disorders or current or prior diagnosis of bulimia or anorexia nervosa -This patient's possible food restriction and excessive exercise suggest the possibility of an eating disorder, and increased seizure risk with a higher dosage of bupropion would be of primary concern option A= clozapine; mood stabilizing agents carbamazepine and valproate can also be associated with bone marrow abnormalities option B= SNRI venlafaxine option D= SSRIs option E= lamotrigine, anticonvulsant used in bipolar disorder option F= atypical antidepressant mirtazapine; selective alpha-2 antagonist= increase serotonin and NE release; 5-HT2 and 5-HT3 receptor antagonist= increase effect of serotonin on free 5-HT1 receptor, antidepressant effect; H1 antagonist

-MPTP is a toxic substance that induces parkinsonism-like syndrome by destroying dopaminergic neurons in the CNS -Series of experiments demonstrated that MPTP per se is minimally toxic; actually, it is a metabolite of MPTP, produced by MAO type B and called MPP+ that causes neurologic damage -Pretreatment with which agent would minimize damage induced by MPTP consumption? a)Amantadine b)Levodopa c)Levodopa/carbidopa d)Selegiline e)Pergolide

answer: Selegiline -Selegiline is an inhibitor of MAO type B (thus, inhibits formation of MPP+ and damage to dopaminergic neurons); clinically, selegiline is used to delay progression of Parkinson option A= antiviral influenza agent; has dopaminergic activity and possibly anticholinergic action as well; therefore, it may provide moderate improvement in patients with Parkinson; amantadine and anticholinergic side effects: dry mouth, nausea, vomiting, blurring of vision, visual hallucinations, and other mental changes option B= dopamine can't cross BBB; levodopa is the immediate precursor of dopamine; it is taken up by the brain, engulfed in the striatum by cell processes that have cell bodies in the substantia nigra; then in the substantia nigra, L-DOPA is converted to dopamine and released as a neurotransmitter; used in parkinson option C= L-DOPA converted by dopa-decarboxylase in periphery; conversion of dopamine in periphery is associated with nausea and vomiting; carbidopa inhibits this process in periphery option E= dopamine agonist which directly stimulates D2 receptors; provides modest improvement in parkinson symptoms when used as monotherapy; however, modest improvement may be sufficient to delay introduction of levodopa by months or years

48 years man -Concern that his skin is excessively dry, red, and crackled -Washes his hands each time he touches something due to fear of contamination; spends 3-4 hours a day washing hands and showers multiple times a day -Fired from job due to refusing to touch keyboards shared by coworkers -Exam: palms are erythematous with peeling skin -Treatment affects which neurotransmitter? a)Acetylcholine b)Dopamine c)GABA d)Glutamate e)Glycine f)Histamine g)Norepinephrine h)Serotonin

answer: Serotonin -Fear of contamination leading to compulsive washing rituals= OCD -OCD= obsessions or compulsions with most experiencing both -Common themes= contamination obsession with cleaning compulsions; obsession with symmetry and compulsions involving ordering and counting; and fear of harm with checking compulsions (e.g., stove off, doors locked); typically engage in time-consuming rituals (>1 hour/day) that cause significant distress and/or functional impairment; patients with washing compulsions may present with skin conditions -SSRIs are first-line treatment

27 years man -Schizophrenia, comes to ED due to acute muscle spasm in neck that developed several days after treatment with haloperidol -Extremely distressed and unable to turn his head; patient is treated with IV diphenhydramine, which relives symptoms within mins -Recommendations are made to discontinue haloperidol and start 2nd generation antipsychotic quetiapine because of its lower risk of similar dystonic side effects -In addition to D2 receptor antagonism, which other effect? a)Dopamine reuptake inhibition b)Dopamine D2 partial agonist activity c)Monoamine oxidase inhibition d)Serotonin 2A receptor antagonism e)Serotonin and norepinephrine reuptake inhibition

answer: Serotonin 2A receptor antagonism -Patient developed acute dystonic reaction (type of extrapyramidal symptom= EPS) several days after exposure to first-generation antipsychotic (FGA) haloperidol -FGAs have high risk of causing EPS due to their potent dopamine D2 receptor antagonism; management of acute dystonic reaction= treatment with diphenhydramine or benztropine and switching to an agent with lower propensity to producing EPS, such as second-generation antipsychotics (SGA) -In addition to dopamine D2 antagonism, SGAs (e.g., olanzapine, quetiapine) also block serotonin receptors, specifically the 5-HT2A receptor -Compared with FGAs, SGAs block 5-HT2A receptor more potently than D2 receptors and have lower binding affinity at the dopamine receptor site, which is thought to explain the reduced risk of causing EPS -Although SGAs have low risk of EPS, they are associated with greater risk of metabolic adverse effects option A= usually in combination with NE reuptake inhibition, is the mechanism of action of many drugs used to treat ADHD, e.g., methylphenidate, bupropion option B= SGAs, aripiprazole and brexpiprazole option C= usually breaks down monoamines, e.g., dopamine, NE, serotonin; MAO inhibitors, e.g., phenelzine, tranylcypromine block these enzymes, thereby increasing availability of respective monoamines option E= SNRIs, e.g., venlafaxine; and TCAs, e.g., amitriptyline

55 years man -History: major depression treated with sertraline for past 4 months, hypertension controlled with lisinopril, diabetes controlled with diet -Side effect of treatment prescribed for depression? a)Cardiac arrhythmias b)Hypertensive crisis c)Orthostatic hypotension d)Seizures e)Serotonin syndrome f)Sexual dysfunction g)Urinary hesitancy h)Weight gain

answer: Sexual dysfunction -Sertraline is an SSRI frequently used as first-line antidepressant therapy -Compared with older antidepressants (e.g., TCAs), SSRIs have minimal activity at alpha-adrenergic, cholinergic and histaminic receptors (more tolerable and safer in overdose) -SSRIs commonly cause sexual dysfunction, including decreased libido, decreased arousal, anorgasmia in women, increased ejaculation latency in men -Sexual dysfunction can occur in more than 50% of patients receiving SSRIs (should be assessed frequently for sexual dysfunction) option A= TCA overdose option B= MAOIs with tyramine containing foods like cheese option C and G= TCAs option D= slightly increased in SSRIs; mainly TCAs and bupropion option E= SSRI combined with MAOI or other serotonergic drug option H= slightly increased in SSRIs

-New inhaled anesthetic under development -During one of experiments, anesthetic agent is administered at a constant partial pressure while arterial and venous concentrations are monitored -Anesthetic has very large arteriovenous concentration gradient shortly after beginning inhalation; property of new anesthetic? a)Fast equilibrium with the brain b)Low peripheral tissue solubility c)Low peripheral tissue uptake d)Low potency e)Slow onset of action

answer: Slow onset of action -Pathway of anesthetic: Inhaled air= lungs= blood= brain (onset of anesthesia occur when sufficient anesthetic reaches brain) -Arteriovenous concentration gradient is the difference between concentration of gas anesthetic in arterial and venous blood -Solubility of anesthetic in peripheral tissues is a major factor determining the size of arteriovenous gradient -If tissue solubility is high a large amount of anesthetic is taken up from arterial blood, which results in low venous concentration; as a result, saturation of the blood requires further absorption of anesthetic in order to replace that which is absorbed by the peripheral tissue (blood saturation takes longer= brain saturation also delayed= onset of action is slower) -Potent anesthetics have low MACs (alveolar concentration that prevents movement in 50% of patients exposed to noxious stimuli)= not related here (choice D)

60 years man -Found by daughter confused at home; found empty bottle of amitriptyline next to his bed -In emergency department, patient is delirious and says that he sees small animals running around in the corner of the room -Appears flushed, brief seizure and becomes unconscious; BP is low and pulse is high; pupils are dilated and equally reactive to light, and his skin and mucous membranes are dry -Initial ECG shows QRS widening and QTc prolongation; transferred to ICU but dies despite resuscitation attempts; pharmacologic effect that contributed to patient's death? a)Increased antihistamine effect b)Sodium channel inhibition c)Synaptic norepinephrine accumulation d)Synaptic serotonin accumulation e)Uncontrolled presynaptic dopamine release

answer: Sodium channel inhibition -TCAs, such as amitriptyline, inhibit reuptake of NE and serotonin and are used in the treatment of depression; they also inhibit fast sodium channel conduction, slowing down myocardial depolarization and potentially leading to cardiac arrhythmias in overdose; refractory hypotension results from decreased cardiac contractility and direct peripheral vasodilation (from peripheral alpha-1 antagonism); most deaths from TCA overdose are due to cardiac arrhythmias and refractory hypotension -Treatment of TCA overdose involves sodium bicarbonate therapy; NaHCO3 increases serum pH, promoting TCA dissociation from sodium channels and increases extracellular sodium concentration; these effects help overcome sodium channel blockade induced by TCAs option A= increases sedation and appetite but doesn't cause death option C= antidepressant effect; can cause seizures and tremors option D= serotonin syndrome; hyperthermia, autonomic instability, hyperreflexia, myoclonus, diaphoresis; most die from cardiac symptoms not serotonin syndrome option E= TCAs has no effect on it

57 years man -Follow up of Parkinson; no changes in motor symptoms, disease well controlled with medications -His wife is very concerned about change in behavior= recently started gambling (lost large amount of saving); other conditions: major depressive disorder (takes sertraline) -Exam: mild resting hand tremor and minimal rigidity -Responsible for change of behavior? a)Decreased peripheral conversion of levodopa b)Increased norepinephrine levels in synaptic cleft c)Increased serotonin levels in synaptic cleft d)Stimulation of central dopamine receptors e)Upregulation of brain nicotinic acetylcholine receptors

answer: Stimulation of central dopamine receptors -Parkinson is caused by loss of dopaminergic neurons in basal ganglia, resulting in symptoms of bradykinesia, tremor and rigidity; dopaminergic medications used to treat PD have been associated with abrupt onset of impulse control disorders, including pathological gambling, compulsive buying, and compulsive sexual behavior -Dopamine agonists (e.g., pramipexole) as a class are more frequently associated with this adverse effect; these drugs act by binding central dopamine receptors and stimulating dopamine activity in the ventral striatum (includes the nucleus accumbens, which is involved in motivation and reward pathways) and substantia nigra option A= mechanism of carbidopa; decreases peripheral symptoms like nausea and vomiting option B= mechanisms of atomoxetine, nonstimulant medication used in treatment of ADHD option C= SSRIs= decrease impulsivity and treat depression and anxiety option E= neuroadaptive response to receptor desensitization that occurs with chronic repeated exposure to high levels of nicotine

27 years woman -Severe, unilateral, throbbing headaches that occur several times a month (associated with photophobia, nausea, and occasional vomiting); over the counter analgesics don't help -Mother has history of similar headaches -Medication prescribed to treat her condition and the patient is instructed to take it immediately at onset of a headache -Medication most likely decreases the severity and duration of this patient's headache through which mechanism? a)Blockade of dopamine receptors in mesolimbic tract b)Blockade of serotonin reuptake in cortico-amygdala circuitry c)Increased availability of acetylcholine in cortical synapses d)Stimulation of periaqueductal gray area mu receptors e)Stimulation of trigeminovascular serotonin receptors

answer: Stimulation of trigeminovascular serotonin receptors -Migraines are recurrent, debilitating, unilateral headaches with a pulsating or throbbing quality lasting between 4-72 hours, often with photophobia, phonophobia or nausea; they can be associated with an aura and are more common in younger women -Pathogenesis (not fully understood): associated with cortical spreading depression (i.e., self-propagating neurotransmission across the cortex); subsequent stimulation of trigeminal afferents in the meninges causes the release of vasoactive neuropeptides, including substance P and calcitonin gene-related peptide (CGRP), leading to neurogenic inflammation, vasodilation, and plasma protein extravasation; these substances may also contribute to the increased sensitivity of nociceptive pathways to stimuli (i.e., neuronal sensitization) -Triptans (e.g., sumatriptan) are serotonin 5-HT 1B/1D agonists that directly counter migraine headaches by binding to trigeminal serotonin receptors and inhibiting CGRP release from trigeminal neurons; in addition, triptans bind serotonin receptors on smooth muscle cells of blood vessels, resulting in intracranial vasoconstriction -Triptans are commonly prescribed as abortive therapy for acute migraines, particularly in patients who are not responsive to analgesics; adverse effects: dizziness, chest tightness and hypertension option A= antipsychotic medications (e.g., haloperidol, risperidone) block the dopamine receptors in the mesolimbic tract and are used in schizophrenia or bipolar disorder option B= SSRIs (e.g., sertraline, citalopram) block serotonin reuptake in cortico-amygdala pathways; used to treat depression and anxiety option C= acetylcholinesterase inhibitors (e.g., rivastigmine, galantamine) are indicated in treatment of dementia, e.g., Alzheimer option D= opioid medications (e.g., morphine, oxycodone) stimulate mu receptors in the periaqueductal gray area, resulting in modulation of pain; opioids are not indicated in treatment of migraines because they increase risk of chronic migraine, carry a risk of dependence, and are less effective than triptans

65 years woman -Pain at posterior chest wall; seen 4 months ago for acute shingles involving a right mid-thoracic dermatome -Not treated at that time because she had delayed presentation and lesions were already well-crusted and healing= lesions cleared completely later -Since then= persistent burning pain and stabbing pain in same region (now more severe) -Tried oral acetaminophen without relief; doesn't want systemic medications due to side effects; started on topical capsaicin cream= pain improves over next several days -Decreased activity of which neurotransmitters is associated with pain relief? a)Acetylcholine b)Dynorphin c)Endorphin d)Glycine e)Histamine f)Neuropeptide Y g)Norepinephrine h)Substance P

answer: Substance P -Shingles is due to reactivation of latent varicella zoster virus in sensory ganglia; it causes a vesicular rash and acute neuritis due to injury to nerves -Initial pain fades within a few weeks to months, but a subset of patients will have residual neuropathic pain in affected dermatome (postherpetic neuralgia, relates to sensitization of afferent nerves in dorsal horn) -It may last several months or longer, and has a burning, stabbing, or gnawing character; PHN is usually managed with TCAs (e.g., amitriptyline) or anticonvulsants (e.g., gabapentin, pregabalin), but nonsystemic options are available (lidocaine patches or topical capsaicin) -Capsaicin is an irritant found in the chili pepper family; it causes excessive activation of TRPV1 (transmembrane cation channel) causing a buildup of intracellular calcium that results in long-lasting dysfunction of nociceptive nerve fibers (defunctionalization); in addition, capsaicin causes release and subsequent depletion of substance P, a polypeptide neurotransmitter involved in transmission of pain signals; on initial application, it causes burning, stinging and erythema, but persistent exposure leads to a moderate reduction in pain over time option B= pain relief by activating kappa opioid receptors option C= pain relief by activating mu opioid receptors option F= found in CNS and ANS; plays role in appetite and pain perception; NPY antagonists investigated as potential antiobesity drugs

45 years man -Quadriplegic, suspected bacterial pneumonia (admitted to hospital and started on IV antibiotics) -Over next 24 hours= develops progressive respiratory failure requiring mechanical ventilation; prior to intubation, skeletal muscle relaxant is administered and patient goes into cardiac arrest -Cardiac monitor shows ventricular fibrillation; while being resuscitated, his serum potassium level is drawn and later comes back as hyperkalemia; administration of which of the following agents is responsible for condition? a)Atracurium b)Baclofen c)Dantrolene d)Succinylcholine e)Vecuronium

answer: Succinylcholine -Succinylcholine is a depolarizing neuromuscular blocking agent that, like Ach, attaches to nicotinic Ach receptor and depolarizes neuromuscular endplate; unlike Ach, succinylcholine is not degraded by acetylcholinesterase, resulting in continuous stimulation of endplate (reflected by initial transient fasciculations) -However, sodium channels surrounding end plate rapidly become inactivated and cannot reopen until end plate is repolarized; thus, succinylcholine-induced depolarization remain isolated to the end plate, resulting in development of flaccid paralysis (phase 1 block); eventually, with continued administrated of succinylcholine, the continuous depolarization of endplate gives way to gradual repolarization as the nAChR becomes desensitized to effects of succinylcholine= phase II block and similar to non-depolarizing blockade -Because nAChR is a nonselective cation channel, its reopening not only allows sodium influx but also potassium release; exaggerated hyperkalemia and life-threatening arrhythmias can occur in patients with crush or burn injuries, denervating injuries or diseases (e.g., quadriplegia, and Guillain-Barre syndrome), and myopathies -These pathologic states cause up-regulation of muscle nAChRs and/or rhabdomyolysis, which can result in release of large amounts of potassium when succinylcholine is administered option A and E= nondepolarizing muscle blockers are better options in these patients option B= muscle relaxant that affects GABA-B receptors at level of spinal cord, used in spasticity, e.g., cerebral palsies option C= muscle relaxant effective in malignant hyperthermia; it acts on ryanodine receptors on SR and prevents calcium release into cytoplasm of muscle fibers

26 years man -History: excessive daytime sleepiness due to narcolepsy enrolls in clinical trial investigating new stimulant medication -Previously treated with modafinil (symptoms didn't improve) -New drug works by promoting release of dopamine and norepinephrine from nerve terminals in the CNS; side effects: primarily due to excess release of NE from peripheral autonomic nerve terminals -Which of the following autonomic efferents would be most affected by use of drug? a)Parasympathetic output to the bronchi b)Parasympathetic output to the heart c)Preganglionic sympathetic output d)Sympathetic output to adrenal glands e)Sympathetic output to eccrine sweat glands f)Sympathetic output to the detrusor muscle

answer: Sympathetic output to the detrusor muscle -In the PSNS, preganglionic neurons arise from cranial nerve nuclei (e.g., CN III, VII, IX, X) and from sacral spinal cord= release Ach= binds postganglionic nicotinic receptors found within the PSNS ganglia in or near walls of target organs; postganglionic neurons also release Ach= activating muscarinic receptors within target organs (e.g., bronchi, heart) -In the SNS, preganglionic neurons arise from thoracolumbar spinal cord and release Ach, which binds postganglionic nicotinic receptors in sympathetic chain and prevertebral ganglia; postganglionic neurons then release NE, activating alpha/beta receptors within target organs (including bladder) (2 exceptions) -The chromaffin cells of the adrenal gland release NE and Epi directly into circulation after stimulation by Ach release from sympathetic preganglionic neurons -Preganglionic and postganglionic sympathetic neurons that supply eccrine sweat glands are both cholinergic

56 years man -History: severe depression, seeks new physician -Depression has been treated with phenelzine since diagnosis, new physician decides to switch him to sertraline -Must wait at least 2 weeks after discontinuing phenelzine before starting new drug -Which of the following processes occurs during washout interval to allow sertraline therapy to be initiated safely? a)G protein-coupled receptor internalization b)Inositol monophosphate inhibition c)Monoamine oxidase inhibition d)Monoamine receptor downregulation e)Monoamine reuptake inhibition f)Synthesis of monoamine oxidase

answer: Synthesis of monoamine oxidase -MAO is an enzyme located in presynaptic nerve terminals that is responsible for breakdown of monoamines (e.g., DA, NE, 5HT) -Phenelzine, a MAOI, works by irreversibly binding and inhibiting both types of MAO (A and B); results in increased presynaptic availability of monoamine neurotransmitters, thereby increasing their release into synaptic cleft -Because phenelzine irreversibly inhibits MAO, it may take up to 2 weeks following discontinuation of the drug before MAO is resynthesized to a level adequate for normal monoamine degradation -SSRIs (e.g., sertraline) increases synaptic serotonin levels; coadministration of SSRIs and MAOIs is contraindicated due to risk of serotonin syndrome (fatal; altered mental status, hyperreflexia, clonus, and autonomic hyperactivity like hyperthermia, tachycardia, diaphoresis); patients must wait 2 weeks to avoid this option A= monoamine receptors work this way option B= mechanism of lithium option C= MAOIs mechanism of action option D= MAOIs mechanism of action option E= Antidepressants work this way

24 years man -Injury to his shoulder (comes to ED); standing on ladder hanging pictures and lost balance and fell, impacting left shoulder on floor -Radiographs= anterior dislocation of humerus; reduction of patient's dislocated joint is performed under procedural sedation with IV propofol (provides sedation for procedure, and becomes clinically alert several mins after administration) -Mechanism that explains patient's rapid recovery from anesthesia? a)Liver elimination of the drug b)Lung elimination of the drug c)Renal elimination of the drug d)Spontaneous degradation in plasma e)Tissue redistribution of the drug

answer: Tissue redistribution of the drug -Propofol is a highly lipophilic anesthetic drug used for both induction and maintenance of general anesthesia as well as procedural sedation -When administered as an IV bolus, its onset of action is approximately 30 secs with a duration of action typically lasting <10 mins; administration causes a rapid decrease in level of consciousness as well as global amnestic effect -Following bolus infusion, propofol is rapidly cleared from plasma and preferentially distributed to organs receiving high blood flow (e.g., brain); this initial distribution accounts for the rapid onset of sedation seen with its administration; over time, propofol is redistributed to organs receiving less blood flow (e.g., fat, muscle tissue); because site of action for propofol is the brain, redistribution accounts for rapid termination of drug action option A= propofol is metabolized by liver through hepatic conversion, its half-life is 4 hours to 1 day; therefore, hepatic metabolism does not occur fast enough to account for such a short duration of action option B= volatile, inhaled anesthetics such as halothane are predominantly eliminated by the lungs option C= renal tubular secretion and glomerular filtration are the predominant mechanisms through which water-soluble drugs are eliminated option D= atracurium is a non-depolarizing neuromuscular blocking agent that spontaneously degrades in plasma and organ tissue, a type of metabolism known as Hofmann elimination

32 years woman -Severe headache, nausea, vomiting (several hours) -Headache is pounding and localized to left side; bright spots in her vision prior to headache onset -Similar episodes since adolescence; exam: normal pupils and fundi, but patient has discomfort to bright light -Treated with metoclopramide and diphenhydramine; why use diphenhydramine? a)To decrease the risk of arrhythmia b)To increase the prokinetic effect c)To prevent abnormal motor reaction d)To prevent allergic reaction e)To prevent rebound headache

answer: To prevent abnormal motor reaction -Normal neurologic exam has unilateral headache associated with nausea and vomiting, photophobia, and visual aura (e.g., flashing lights)= migraine headache -Severe migraine headaches with nausea and vomiting is typically treated with serotonin 1B/1D agonist (i.e., triptans) or a dopamine receptor blocker (e.g., metoclopramide, prochlorperazine) to abort the migraine and treat nausea -Dopamine receptor blockers have both antiemetic and analgesic effects on migraine headaches by antagonizing central dopamine D2 receptors in the midbrain and striatum -Dopamine receptor blockade in the basal ganglia can also cause excess cholinergic activity that manifests as significant extrapyramidal symptoms, such as acute dystonic reaction (e.g., torticollis, oculogyric crisis) -Therefore, diphenhydramine can be co-administered to prevent such reactions due its anticholinergic activity option A= 5HT3 receptor antagonists ondansetron and granisetron have a low risk of QT prolongation leading to torsades de pointes option B= metoclopramide has prokinetic effect (e.g., enhances gastric emptying) and can be useful in treating nausea and vomiting associated with diabetic gastroparesis; adding diphenhydramine would decrease this effect due to anticholinergic activity option D= diphenhydramine can be used to treat urticaria, but not prevent recurrence option E= e.g., >10 days a month) of certain abortive migraine medications (e.g., triptans, analgesics), but not dopamine antagonists

26 years man -Severe headaches, increasing in frequency; episodic, right-sided hemicranial throbbing or pulsating headaches accompanied by nausea and photophobia (started in high school, now more frequent) -Takes NSAIDs and sumatriptan for pain and metoclopramide for nausea -Everything else normal (exam, CT, etc.); pharmacotherapy that would help prevent recurrence of headaches? a)Capsaicin b)Ergotamine c)Hydrochlorothiazide d)Sertraline e)Topiramate

answer: Topiramate -Patient has recurrent migraine (severe, unilateral headaches, frequently associated with nausea and sensitivity to light and sound/aura) -Abortive therapy= for acute pain control; preventative therapy= reduce frequency and severity of headaches -Abortive medications: mild analgesics (NSAIDs, acetaminophen), triptans (e.g., sumatriptan), antiemetics (e.g., metoclopramide, prochlorperazine) and ergotamines= reduce symptoms of headache onset, do not prevent -Preventive therapy: topiramate (anticonvulsant), beta blockers (e.g., metoprolol, propranolol), TCAs (e.g., amitriptyline), and other anticonvulsants (e.g., valproate) option A= derived from chili peppers and is useful as topical therapy for neuropathic or musculoskeletal pain option B= agonist and antagonist on multiple neurotransmitters (e.g., dopamine, serotonin, epinephrine) and acts as a vasoconstrictor; used for migraine abortion option C= antihypertensive, reduces sodium reabsorption in DCT; not useful in migraines option D= SSRI; used for depression and anxiety; no help in migraines

34 years man -Brought to ED by wife after she found him running around the yard in his underwear shouting "I am going to change the world"; he has lots of energy despite hardly sleeping for past 7 days, and recently quit his job to write a 500-page novel that he says "explains everything about the universe" -History: significant for epilepsy with generalized seizures since childhood and psychiatric hospitalization a year ago for major depressive episode -Family history: alcoholism, hypertension in father and schizophrenia in grandmother -Medication helpful for long-term treatment of his psychiatric and seizure disorders? a)Aripiprazole b)Citalopram c)Levetiracetam d)Lithium e)Lorazepam f)Phenytoin g)Valproate

answer: Valproate -Patients history of major depression along with his current manic symptoms (i.e., decreased need for sleep, grandiosity, goal-directed activity) indicate diagnosis of bipolar disorder -Valproate (valproic acid) is used as an anticonvulsant and mood stabilizer -it blocks voltage-gated sodium channels and enhances availability of GABA; as a mood stabilizer, it is used in acute mania and maintenance treatment of bipolar disorder; as an anticonvulsant, it effectively treats absence, myoclonic, and generalized tonic-clonic seizures -Drug of choice for myoclonic seizures and second-line for absence seizures (for which ethosuximide is preferred) -Other anticonvulsants used as mood stabilizers in bipolar include carbamazepine and lamotrigine; carbamazepine is a CYP450 inducer that can cause lower levels of concomitant medications; lamotrigine can cause Stevens Johnson syndrome option A= SGA used to treat psychotic and mood disorders; antipsychotics may lower seizure threshold option B= SSRI for depression; avoided in bipolar as monotherapy due to risk of inducing mania option C and F= anticonvulsants used in treatment of epilepsy, not useful in bipolar disorder option D= mood stabilizer that is effective in bipolar disorder; modifies second messenger signal transduction (e.g., adenylyl cyclase and phospho-inositol pathways) at several levels; resulting in decreased excitatory neurotransmission; not anticonvulsant option E= BZ; used for active seizures, acute anxiety and alcohol withdrawal

15 years boy -3 months of sudden, brief jerking movements involving both arms -Episodes usually occur shortly after awakening in morning and aggravated by sleep deprivation -Movements neither suppressible nor preceded by an urge to make a movement; patient neve lost consciousness and no known medical conditions; family history= seizure disorder in uncle -Best initial treatment? a)Baclofen b)Carbamazepine c)Clonidine d)Diazepam e)Propranolol f)Valproic acid g)Tetrabenazine

answer: Valproic acid -Patient has myoclonic seizures due to juvenile myoclonic epilepsy (JME), a form of generalized onset epilepsy (i.e., affecting both cerebral hemispheres at onset) that may be familial; typically, JME manifests as an otherwise healthy adolescent with myoclonic seizures (i.e., brief, involuntary jerking movements) involving both upper extremities with preservation of consciousness -Symptoms usually occur within the first hour of awakening and can be provoked by sleep deprivation; patients can also develop tonic-clonic and/or absence seizures -Treatment of generalized onset epilepsies like JME should include a broad-spectrum anticonvulsant (e.g., valproic acid, levetiracetam) which can treat both focal ang generalized seizures; in contrast, narrow-spectrum anticonvulsants (e.g., carbamazepine, phenytoin) are primarily used in patients with focal onset seizures and can worsen seizure symptoms in patients with absence or myoclonic seizures (choice B) option A= GABA-B agonist that can treat chronic muscle spasticity (e.g., cerebral palsy); patients with chronic muscle spasticity typically have progressive changes in development and muscle tone that would be apparent on physical examination option C= alpha-2 agonist that may be effective in children with ADHD who have failed treatment with stimulants, e.g., methylphenidate option D= BZ that can be used acutely to treat prolonged seizures such as status epilepticus (>5 mins of continuous seizure activity or >1 seizure without recovery in between option E= nonselective beta blocker that can treat benign essential tremor, which typically presents in middle-aged adults with bilateral, rhythmic hand tremor that worsens during fine motor activity option G= dopamine depletion agent that can treat Tourette syndrome, disorder characterized by motor and phonic tics that persist for >1 year; motor tics can manifest as periodic, sudden, jerking movements, but they are usually temporarily suppressed and often preceded by an urge to make the stereotyped movement

11 years boy -Brought to ED after briefly losing consciousness; mother found him on kitchen floor shaking and jerking (was out of her sight for few mins) -Says son often stares into space and doesn't respond to questions during these episodes -No known medica; conditions or history of illnesses; patient seems sleepy but is oriented time and place -Several hours later= alert, wants to go home; best long-term treatment for this patient? a)Carbamazepine b)Ethosuximide c)Gabapentin d)Phenytoin e)Valproic acid

answer: Valproic acid -Patient with frequent staring spells (i.e., absence seizures) and an unprovoked tonic-clonic seizure most likely has undiagnosed juvenile absence epilepsy -Absence seizures are characterized by momentary lapses in consciousness (10 seconds) associated with a classic 3-Hz spike-wave pattern on EEG; younger children with absence seizures typically do not develop other seizure types; however, later onset (e.g., age 10-12) of absence epilepsy is associated with increased incidence of generalized onset tonic-clonic or myoclonic seizures -First-time unprovoked seizure may not require treatment, this patient had multiple seizures, including both absence and tonic-clonic seizures -Ethosuximide is first-line for treatment of isolated absence seizures; however, it doesn't suppress tonic-clonic seizures (choice B) -Treatment with broad-spectrum antiepileptic such as valproic acid is required in these patients because it is effective for both absence and tonic-clonic seizures option A, C, D= narrow-spectrum antiepileptics used to treat focal-onset seizures, including focal onset seizures that progress to bilateral tonic-clonic seizures

42 years woman -Surgical removal of mediastinal mass (hospitalized) -Had intermittent episodes of double vision, difficulty chewing food, weakness of extremities for past several months -Evaluation: circumscribed anterior mediastinal mass -Preoperative nerve conduction studies= decremental response to repetitive motor nerve stimulation; no allergies to drugs; compared to healthy individual, which of the following anesthesia-related medications is most likely to have increased potency in this patient? a)Isoflurane b)Midazolam c)Neostigmine d)Atropine e)Vecuronium

answer: Vecuronium Patient has anterior mediastinal mass and nerve conduction study is consistent with myasthenia gravis (autoimmune disease caused by autoantibodies against postsynaptic nicotinic Ach receptors of the neuromuscular junction) Antibody-induced degradation leads to fewer functional receptors, which may be insufficient to general muscle action potentials despite normal Ach release Weakness (most commonly of the extraocular, bulbar, and proximal muscles) worsens with repetitive nerve stimulation (fatigability) as Ach stores in the presynaptic nerve terminals are depleted; Thymic abnormalities (e.g., thymoma, thymic hyperplasia) are common and may appear as anterior mediastinal mass on imaging Neuromuscular blocking agents are used to induce skeletal muscle relaxation during surgery by competing with Ach at the postsynaptic membrane; they are subclassified by their effects: -Nondepolarizing agents (e.g., vecuronium, rocuronium) are competitive antagonists of nicotinic Ach receptors; due to low number of functioning receptors, patients with MG are extremely sensitive to these agents, very small doses can induce paralysis with loss of airway resistance -Depolarizing agents (e.g., succinylcholine) are competitive agonists of nicotinic Ach receptors; persistent depolarization leads to desensitization of motor endplate; due to low number of functioning receptors, patients with MG are often resistant to these agents, extremely large doses required to induce muscle relaxation option A= general anesthetic (acts centrally on multiple receptors) that can cause malignant hyperthermia in those with genetic predispositions; contraindicated in MG option B= BZ (acts on GABA-A receptor) to produce sedation; contraindicated in MG option C= mainstay symptomatic treatment for MG is acetylcholinesterase inhibitors (e.g., pyridostigmine, neostigmine), these agents inhibits degradation of Ach in synapse option D= muscarinic Ach receptor antagonist often used as premedication before general anesthesia to decrease salivary and bronchial secretions (helps maintain airway patency) and prevent myocardial depression (minimizes risk of hypotension), no effect on nicotinic Ach receptors

-Elderly immigrant from the Middle East presents with numbness and tingling of hands and feet -Diagnosed with pulmonary tuberculosis several months ago and currently receiving isoniazid and rifampin -Exam: sensory ataxia and decreased pain sensation in distal extremities -Caused by? a)immune-mediated demyelination b)Bacterial toxin-mediated axonal damage c)Drug-mediated toxic neuron loss d)Vitamin deficiency e)Diabetic neuropathy

answer: Vitamin deficiency -Isoniazid is chemically similar to pyridoxine (vitamin B6); isoniazid can compete with vitamin B6 in the synthesis of multiple neurotransmitters (including GABA)= defective end products -Isoniazid also increases the urinary excretion of pyridoxine, and a frank deficiency in this vitamin can develop -Vitamin B6 deficiency= with isoniazid (in elderly, alcohol use disorder, or with other comorbidities like liver or kidney dysfunction) -Manifests as peripheral neuropathy= supplementation of vitamin B6 is required with isoniazid option A= occurs in multiple sclerosis and Guillain-Barre syndrome option B= infections secondary to Corynebacterium diphtheria, a microorganism that causes pseudomembranous pharyngitis; C. diphtheria synthesizes potent exotoxin associated with cardiac and neural toxicity option C= chemotherapeutic agents that inhibit microtubule formation; e.g., vincristine and paclitaxel

14 years boy -Brought by mother for seizures; characterized as a blank stare and lip smacking, followed by generalized shaking -Occasionally bites tongue during episodes and feels confused afterwards -Seizures have been refractory to various antiepileptic medications= started on lamotrigine -Patient should be instructed to see doctor immediately after? a)Abdominal pain b)Cough c)Diarrhea d)Heartburn e)Skin rash f)Urine discoloration

answer: skin rash -Lamotrigine (anticonvulsant)= blocks voltage-gated Na channels (used to treat partial and generalized seizures, also effective for bipolar disorder) -Up to 10% of patients using lamotrigine will develop benign rash; however, a potentially life-threatening rash due to Stevens Johnson syndrome or toxic epidermal necrolysis occurs in 1% of patients (particularly children); SJS and TEN represent different endpoints along the same disease continuum: SJS involves <10% of body surface area, and TEN has >30% involvement -Patients with SJS/TEN typically develop flu-like symptoms (e.g., fever, malaise, myalgias) followed by extensive cutaneous and mucosal lesions; histology= epidermal necrosis and subepidermal bullae -First appearance of rash requires discontinuation of drug immediately as SJS/TEN and benign rash can't be distinguished reliably; other anticonvulsants associated with SJS/TEN include carbamazepine, phenobarbital, phenytoin -GI symptoms are common (abdominal pain, heartburn, diarrhea) with lamotrigine or other anticonvulsants= but non-life-threatening


संबंधित स्टडी सेट्स

Ch. 19 Cardiovascular System: Blood Vessels A&P II

View Set